biochem.zsmu.zp.ua · web viewe. monosaccharides the rate limited step for citric acid cycle...

111
Name the metabolic pathway which is in need for vitamins B<sub>1</sub>, B<sub>2</sub>, B<sub>3</sub>, B<sub>5</sub> supplement at the same time: A. Krebs Cycle B. Pentose Phosphate Cycle C. Urea Cycle D. Glycogenesis E. Glycolysis Propose the main food product to promote the intake of vitamin C: A. Butter B. Beef C. Milk D. Parsley E. Black currant Name the blood plasma index whose low value will prove the deficiency of vitamin K in patient: A. Urea B. Prothrombin C. Immunoglobulin G D. C-reactive protein E. Albumins A 10-year-old girl often experiences acute respiratory infections with multiple hemorrages in the places of clothes friction. Hypovitaminosis of what vitamin is in this girl organism? A. B<sub>6</sub> B. С C. B<sub>2</sub> D. A E. B<sub>1</sub> A patient presents with twilight vision impairment. Which of the following vitamins should be administered? A. Nicotinic acid B. Cyanocobalamin C. Pyridoxine hydrochloride D. Retinol acetate E. Ascorbic acid A patient presents with twilight vision impairment. Which of the following vitamins should be administered? A. Nicotinic acid B. Retinol acetate C. Cyanocobalamin D. Pyridoxine hydrochloride E. Ascorbic acid

Upload: lekiet

Post on 11-May-2018

216 views

Category:

Documents


0 download

TRANSCRIPT

Name the metabolic pathway which is in need for vitamins B<sub>1</sub>, B<sub>2</sub>, B<sub>3</sub>, B<sub>5</sub> supplement at the same time:

A. Krebs CycleB. Pentose Phosphate CycleC. Urea CycleD. GlycogenesisE. Glycolysis

Propose the main food product to promote the intake of vitamin C:

A. ButterB. BeefC. MilkD. ParsleyE. Black currant

Name the blood plasma index whose low value will prove the deficiency of vitamin K in patient:

A. UreaB. ProthrombinC. Immunoglobulin GD. C-reactive proteinE. Albumins

A 10-year-old girl often experiences acute respiratory infections with multiple hemorrages in the places of clothes friction. Hypovitaminosis of what vitamin is in this girl organism?

A. B<sub>6</sub>B. СC. B<sub>2</sub>D. AE. B<sub>1</sub>

A patient presents with twilight vision impairment. Which of the following vitamins should be administered?

A. Nicotinic acidB. CyanocobalaminC. Pyridoxine hydrochlorideD. Retinol acetateE. Ascorbic acid

A patient presents with twilight vision impairment. Which of the following vitamins should be administered?

A. Nicotinic acidB. Retinol acetateC. CyanocobalaminD. Pyridoxine hydrochlorideE. Ascorbic acid

Choose the name of phase for vitamin K use in the formation of clotting factors:

A. ReparationB. TranscriptionC. ReplicationD. Post-transcriptionE. Post-translation

A patient suffers from vision impairment – hemeralopia (night blindness). What vitamin preparation should be administered the patient in order to restore his vision?

A. Retinol acetateB. Tocopherol acetateC. PyridoxineD. Thiamine chlorideE. Vicasol

Choose the vitamin, whose antivitamin is named as Dicoumarol:

A. Vitamin KB. Vitamin B<sub>6</sub>C. Vitamin DD. Vitamin AE. Vitamin C

A patient with hypochromic anemia has splitting hair and loss of hair, increased nail bottling and taste alteration. What is the mechanism of the development of these symptoms?

A. Decreased production of parathyrinB. Deficiency of iron-containing enzymesC. Decreased production of thyroid hormonesD. Deficiency of vitamin AE. Deficiency of vitamin B<sub>12</sub>

Choose the vitamin, whose deficiency leads to osteomalacia at adults:

A. Vitamin DB. Vitamin KC. Vitamin PPD. Vitamin EE. Vitamin C

A patient who was previously ill with mastectomy as a result of breast cancer was prescribed radiation therapy. What vitamin preparation has marked radioprotective action caused by antioxidant activity?

A. ErgocalciferolB. RiboflavinC. Thiamine chlorideD. Tocopherol acetateE. Folic acid

After the disease a 16-year-old boy is presenting with decreased function of protein synthesis in the liver as a result of vitamin K deficiency. This may cause disorder of:

A. Osmotic blood pressureB. Anticoagulant productionC. Erythropoietin productionD. Blood coagulationE. Erythrocyte sedimentation rate

Choose the vitamin which is important in non-oxidative decarboxylation, transamination and transsulfuration reactions from the following list:

A. RiboflavinB. PyridoxineC. Pantothenic acidD. ThiamineE. Folic acid

Name the vitamin whose level in the blood is depended on the secretion rate of parathyroid hormone:

A. Ascorbic acidB. ThiamineC. Naphtoquinone

D. CalcitriolE. Tocopherol

Point out vitamin which is the most indispensable during mitosis:

A. Ascorbinic acidB. Pantotenic acidC. Folic acidD. Aspartic acidE. Thiamine

Find out the Enzyme name whose activity is depended on the presence of vitamin B<sub>2</sub>:

A. Malate dehydrogenaseB. Citrate synthaseC. Isocitrate dehydrogenaseD. Succinate dehydrogenaseE. Pyruvate carboxylase

Choose the vitamin, which is a powerful natural antioxidant:

A. TocopherolB. RiboflavinC. RetinalD. ErgocalciferolE. Pyridoxine

Find out the fat-soluble vitamin whose function is hormone-similar one:

A. Vitamin PPB. Vitamin DC. Vitamin KD. Vitamin CE. Vitamin E

The avitaminosis of ascorbic acid is named as:

A. ScurvyB. Cushing`s syndromeC. Addison`s diseaseD. KwashiorkorE. Hemolytic anemia

Choose the vitamin, whose molecule structure is unsaturated cyclic alcohol (one hydroxide-group only):

A. Vitamin D<sub>2</sub>B. Vitamin HC. Vitamin KD. Vitamin FE. Vitamin B<sub>5</sub>

Choose the most active form of vitamin D<sub>3</sub>:

A. 24,25-DihydroxycholecalciferolB. CalcidiolC. 25-HydroxycholecalciferolD. 25-HydroxyergocalciferolE. 1,25-Dihydroxycholecalciferol

What reaction is in need of vitamin K?

A. OxidationB. Hydroxylation

C. Alpha-decarboxylationD. MethylationE. Gamma-carboxylation

Examination of a man who hadn’t been consuming fats but had been getting enough carbohydrates and proteins for a long time revealed dermatitis, poor wound healing, vision impairment. What is the probable cause of metabolic disorder?

A. Low caloric value of dietB. Lack of palmitic acidC. Lack of linoleic acid, vitamins A, D, E, K.D. Lack of vitamins PP, HE. Lack of oleic acid

Choose the correct definition of vitamin:

A. Food factors that cannot be synthesized in human organismB. Essential biologic aminesC. Organic compounds, containing amino groupD. Essential food proteinsE. Essential energy sources

Point out the vitamin, whose deficiency leads to pellagra:

A. Vitamin CB. Vitamin B<sub>2</sub>C. Vitamin PD. Vitamin B<sub>3</sub>E. Vitamin A

Choose the vitamin that contains the isoalloxazine fragment in its structure:

A. NaphtoquinoneB. UbiquinoneC. RiboflavinD. ThiamineE. Pyridoxine

Concentration of pyruvate is increased in the patient's blood, the most of which is excreted with urine. What avitaminosis has the patient?

A. Avitaminosis B<sub>3</sub>B. Avitaminosis B<sub>9</sub>C. Avitaminosis ED. Avitaminosis B<sub>2</sub>E. Avitaminosis B<sub>1</sub>

Vitamin A group contains substance whose function is associated mainly with stimulation of proliferation and differentiation processes in tissues. Name it:

A. RetinalB. Retinoic acidC. Nicotine amideD. Pantothenic acidE. Nicotinic acid

Which vitamin is related to cofactor in glycine metabolism?

A. ThiamineB. Folic acidC. Pantothenic acidD. CobalaminE. Tocopherol

Choose the vitamin, whose precursor is named as &beta;-carotene:

A. Vitamin PB. Vitamin B<sub>12</sub>C. Vitamin CD. Vitamin DE. Vitamin A

Hydroxylation of endogenous subrates and xenobiotics requires a donor of protons. Which of the following vitamins can play this role?

A. Vitamin B<sub>6</sub>B. Vitamin PC. Vitamin CD. Vitamin EE. Vitamin A

Examination of a patient with frequent hemorrhages from internals and mucous membranes revealed proline and lysine being a part of collagen fibers. What vitamin absence caused disturbance of their hydroxylation?

A. Vitamin KB. Vitamin CC. Vitamin ED. Vitamin AE. Thiamine

On the ground of clinical presentations a patient was prescribed pyridoxal phosphate. This medication is recommended for correction of the following processes:

A. Desammation of purine nucleootidesB. Protein synthesisC. Transamination and decarboxylation of amino acidsD. Synthesis of purine and perimidin basesE. Oxidative decarboxylation of ketonic acids

Point out the vitamin, which is soluble in lipids:

A. Vitamin B<sub>1</sub>B. Vitamin HC. Vitamin KD. Vitamin CE. Vitamin PP

The structural analogue of vitamin B<sub>2</sub> is administered (acriсhine) in a case of enterobiasis. The disorder of which enzyme synthesis is caused by this medicine in microorganisms?

A. PeptidasesB. Cytochrome oxidasesC. NAD-dependent dehydrogenasesD. AminotransferasesE. FAD-dependent dehydrogenases

There is disturbed process of Ca2+ absorption through intestinal wall after the removal of gall bladder in patient:

A. CB. B12C. D<sub>3</sub>D. KE. PP

A woman who has been keeping to a clean-rice diet for a long time was diagnosed with polyneuritis (beriberi). What vitamin deficit results in development of this disease?

A. Folic acidB. RiboflavinC. ThiamineD. PyridoxineE. Ascorbic acid

A 9-month-old infant is fed with artifical formulas with unbalanced vitamin B<sub>6</sub> concentration. The infant presents with pellagral dermatitis, convulsions, anaemia. Convulsions development might be caused by the disturbed formation of:

A. GABAB. DOPAC. HistamineD. SerotoninE. Dopamine

During examination of an 11-month-old infant a pediatrician revealed osteoectasia of the lower extremities and delayed mineralization of cranial bones. Such pathology is usually provoked by the deficit of the following vitamin:

A. ThiaminB. Pantothenic acidC. RiboflavinD. BioflavonoidsE. Cholecalciferol

There is an inhibited coagulation in the patients with bile ducts obstruction, bleeding due to the low level of absorption of vitamin. What vitamin is in deficiency?

A. DB. CaroteneC. KD. AE. E

Choose the vitamin, whose oxidation results in blue fluorescing product under UV-light:

A. Ascorbic acidB. RutinC. Folic acidD. PyridoxineE. Thiamine

The glycolysis duration is in need for one vitamin, only. Name it:

A. ThiamineB. Nicotinic acidC. RiboflavinD. Ascorbic acidE. Pyridoxal phosphate

A 6 y.o child was administered vicasol to prevent postoperative bleeding. Vicasol is a synthetic analogue of vitamin K. Name post-translation changes of blood coagulation factors that will be activated by vicasol:

A. GlycosylationB. PolymerizationC. Partial proteolysisD. Phosphorylation of serine radicalsE. Carboxylation of glutamic acid residues

Most participants of Magellan expedition to America died from avitaminosis. This disease declared itself by general weakness, subcutaneous hemmorhages, falling of teeth, gingival hemmorhages. What is the name of this avitaminosis?

A. ScurvyB. Biermer's anemiaC. RachitisD. Polyneuritis (beriberi)E. Pellagra

Choose the correct statement about vitamin K:

A. It is present in increased amount in cows and breast milkB. It is helpful in preventing thrombosisC. It is synthesized by intestinal bacteriaD. It is derived to coenzyme of &alpha-decarboxylasesE. It increases a coagulation time in infants with hemorrhagic diseases

Pyruvate concentration in the patient's urine has increased 10 times from normal amount. What vitamin deficiency can be the reason of this change:

A. Vitamin AB. Vitamin CC. Vitamin B<sub>6</sub>D. Vitamin B<sub>1</sub>E. Vitamin E

A 2-year-old child has got intestinal dysbacteriosis, which results in hemorrhagic syndrome. What is the most likely cause of hemorrhage of the child?

A. HypocalcemiaB. Vitamin K insufficiencyC. Activation of tissue thromboplastinD. Fibrinogen deficiencyE. PP hypovitaminosis

After the disease a 16-year-old boy is presenting with decreased function of protein synthesis in the liver as a result of vitamin K deficiency. This may cause disorder of:

A. Blood coagulationB. Erythrocyte sedimentation rateC. Erythropoietin productionD. Anticoagulant productionE. Osmotic blood pressure

Find out the vitamin whose deficiency is associated with damaged transamination of amino acids:

A. ThiamineB. Folic acidC. PyridoxineD. RutinE. Ascorbic acid

Different functional groups may be found in the structure of L-amino acid residues of proteins. Identify the group that is able to form ester bond:

A. –SHB. –NH[sub]2[/sub]C. –CH[sub]3[/sub]D. –OHE. –CONH[sub]2[/sub]

The mixture of proteins can be separated by salting-out. Specify the reagent formula that is used for this purpose:

A. C[sub]6[/sub]H[sub]6[/sub]B. HNO[sub]3[/sub]C. H[sub]3[/sub]PO[sub]4[/sub]D. C[sub]2[/sub]H[sub]5[/sub]OHE. NaCl

Histones are related to basic proteins. That is because there is high content of basic amino acid residues in their structure. Point out these amino acids:

A. Аlanine, GlycineB. Arginine, LysineC. Tryptophan, TyrosineD. Asparagine, GlutamineE. Leucine, Valine

Point out the type of bond allowing the formation of alpha-helix structure:

A. Disulfide bond between two cysteine radicalsB. Hydrogen bonds between peptide fragmentsC. Hydrogen bonds between side-chain radicalsD. Electrostatic interactionE. Ester bond between side chain radicals

Proteins are obligatory components of human diet. Specify the function of proteins in this case:

A. TransportB. StructuralC. CatalyticD. NutritiveE. Regulatory

Protonation and deprotonation at neutral pH occurs for one amino acid from following list. Point out it:

A. HistidineB. Aspartic acidC. GlycineD. AlanineE. Tryptophan

The proteins are able to carry out the regulatory function. Find out those protein:

A. InsulinB. Immunoglobulin GC. AminopeptidaseD. CollagenE. Hemoglobin

How many polypeptide chains (the least number) may be represented in quaternary structure of protein molecule?

A. FourB. TwoC. ThreeD. OneE. Five

The yellow color sediment appearance after the addition of strong nitric acid to albumin solution is due to the presence of aromatic acid residues in protein composition. Choose those one:

A. L-alanine

B. L-tyrosineC. L-methionineD. L-serineE. L-proline

Alpha- helix and Beta -pleated sheet are examples of:

A. Primary structureB. Quaternary structureC. Tertiary structureD. All positions are wrongE. Secondary structure

What class of proteins Albumins and Globulins are related to?

A. MetalloproteinsB. GlycoproteinsC. Simple proteinsD. ChromoproteinsE. Lipoproteins

The solubility of proteins in saline solutions is determined by their native structure. Point out the protein, which will swell only in saline solution:

A. ImmunoglobulinB. PepsinC. MyoglobinD. ElastinE. Albumin

All proteins are divided into simple and conjugated ones. Find out the simple proteins among these ones:

A. Albumin of eggB. Globulin of eggC. ProtamineD. HistoneE. All the proteins presented

All are aromatic amino acids EXCEPT:

A. All the positions are correctB. LysineC. TyrosineD. PhenylalanineE. Tryptophan

Protein properties may be changed under the influence of some factors. Find out them:

A. The temperature of the environmentB. Strong acidic mediumC. Strong alkaline mediumD. Organic solventE. All the factors that are present

Polypeptide chains of collagen include specific amino acids. Name one of them:

A. Formyl-methyonineB. HydroxyprolineC. OrnithineD. Beta-alanineE. Cysteine

What physical-chemical property is observed for fibrous protein only?

A. ElasticityB. Amphoteric propertiesC. Solubility in waterD. Solubility in lipidsE. Denaturation and renaturation

Half saturation test using salting-out is done for:

A. FibrinB. GlobulinC. HaemoglobinD. ProthrombinE. Albumin

There are many important protein functions in the human organism. Point out that of them, which isn't peculiar for proteins – to be :

A. AntibodyB. CatalystC. Transfer of substancesD. SolventE. Structural component of a cell

The polypeptide chain gets the globular structure after the formation of various bonds between the radicals of amino acid residues. Specify the strongest bond in the globular structure:

A. Hydrogen bondB. Sodium hydroxideC. Hydrophobic interactionD. Disulfide bondE. Donor-acceptor bondF. Electrostatic interaction

The isoelectric point (I.P.) of a protein depends upon the amino acids composition of the protein. Choose the amino acid which high content in protein molecule decreases its I.P. value:

A. Aspartic acidB. АlanineC. LeucineD. HistidineE. Tryptophan

The tertiary structure of protein is formed mainly due to disulfide bonds between side radicals of one amino acid, only. Point out it:

A. LysB. HisC. MetD. AspE. Cys

Which of the following bonds are intact during denaturation of proteins:

A. Ionic bondsB. All positions are correctC. Peptide bondsD. Hydrogen bondsE. Hydrophobic bonds

Ninhydrin is a reagent to prove the presence of alpha-amino group in the structure of amino acid due to change of its color (violet color is observed). Choose the amino acid whose solution changes the color of this reagent in other way – it becomes yellow:

A. L-methionineB. L-serineC. L-tyrosineD. L-prolineE. L-alanine

Biuret test is mainly done for:

A. Any of the proposedB. PolysaccharidesC. LipidsD. DipeptidesE. Proteins

What type of amino acids mainly is represented as residues in proteins of human body?

A. D-alpha-imino acidsB. L-alpha-amino acidsC. D-beta-amino acidsD. L-beta-amino acidsE. D-alpha-amino acids

Choose the factor that causes the sedimentation of protein in solution without denaturation:

A. TolueneB. Nitric acidC. Sodium hydroxideD. Sulfuric acidE. Ammonia sulfate

The content of these amino acids in the composition of acidic protein Pepsin is too big in comparison with the content of other amino acids in it. Name them:

A. Aspartic acid and Glutamic acidB. Alanine and GlycineC. Lysine and ArginineD. Tyrosine and TryptophanE. Valine and Leucine

Primary structure of proteins is formed due to one type of bonds. Point out it:

A. Metal bondB. Hydrogen bondC. Ester bondD. Peptide bondE. Disulfide bond

The isoelectric point of simple protein equals 7.2. Propose the pH of buffer solution used for the electrophoresis method to separate this protein from the mixture with a condition to leave it on the start line of carrier:

A. pH=7.0B. pH=7.4C. pH=7.6D. pH=7.2E. pH=5.0

What amino acid high content presence in the composition of polypeptide chain does not allow the formation of alpha-helical structure as secondary level of organization?

A. ThreonineB. Proline

C. GlycineD. SerineE. Alanine

The structural proteins are involved in maintaining the shape of a cell or in the formation of matrices in the body. Point out shape of these proteins:

A. GlobularB. All the proposedC. Stretch of beadsD. FibrousE. Planar

The molecule of simple protein insulin contains two polypeptide chains. Specify the level of protein organization, at which the insulin is able to act as hormone:

A. A conformation after limited proteolysisB. SecondaryC. QuaternaryD. PrimaryE. Tertiary

Which method is appropriate for the determination of total protein content in the blood serum:

A. Salting outB. ElectrophoresisC. DialysisD. Fole’s testE. Biuretic method

There is the use of electrophoresis for separation of proteins of blood plasma to prove diagnosis of diseased persons. Name the property of proteins that is the basis for the principle of electrophoresis method:

A. The high viscosity of the solutionB. The net charge of the moleculeC. The big mass of the moleculeD. Optical activityE. The ability of swelling

Toxic affection of liver results in dysfunction of protein synthesis. It is usually accompanied by the following kind of dysproteinemia:

A. Absolute hyperproteinemiaB. Relative hyperproteinemiaC. Absolute hypoproteinemiaD. ParaproteinemiaE. Relative hypoproteinemia

The diseased person with diagnosis acute kidney insufficiency is in urological department of hospital. Choose the method for cleaning of this person`s blood from low-molecular compounds which can cause toxic effect in the organism:

A. ElectrophoresisB. Salting-outC. HydrolysisD. DialysisE. Affine chromotography

Point out a possible cause of hypoproteinemia:

A. ParaproteinemiaB. Affected liver cells

C. OvereatingD. Multiple myelomaE. Decreased permeability of the capillary wall

Flavoproteins are usually catalysts in a cell due to the presence of special vitamin fragment in their structure. Name this vitamin:

A. Nicotin amideB. Folic acidC. RiboflavinD. Ascorbic acidE. Pantothenic acid

Name the location of deoxyribonucleoproteins in eukaryotic cell, but not in the nucleus:

A. Endoplasmic reticulumB. MitochondriaC. LisosomeD. MicrosomeE. Cytoplasma

Albumins of blood plasma are negative charged under the condition of electrophoresis method duration. What electrode type will you choose for albumins directed movement in the electric field?

A. Silk electrodeB. Carbon electrodeC. CatodeD. AnodeE. Calomel electrode

Name the class of proteins used for the formation of ribosome subunits:

A. PhosphoproteinsB. GlycoproteinsC. LipoproteinsD. FlavoproteinsE. Ribonucleoproteins

Triacylglycerols (TG) are synthesized in the liver but are stored in adipose tissue. Name the class of proteins promoted the transport of TG from the liver to adipocytes:

A. MetalloproteinsB. PhosphoproteinsC. LipoproteinsD. GlycoproteinsE. Chromoproteins

Choose the conjugated protein in possession of following characteristics: quaternary structure - 4 polypeptide chains; non-protein part – 4 hemes; function – oxygen transport in the blood:

A. CeruloplasminB. Low Density LipoproteinC. ImmunoglobulinD. AlbuminE. Hemoglobin

It is in need to use the heme with iron ion for the active centers of cytochrome C oxidase (the key enzyme of tissue respiration). Name the class of this conjugated protein:

A. GlycoproteinsB. LipoproteinsC. ChromoproteinsD. Nucleoproteins

E. Flavoproteins

The enzyme preparation cytochrome C is used for the improvement of tissue respiration in newborns at asphyxia state. Name the class of this conjugated protein:

A. ChromoproteinsB. FlavoproteinsC. NucleoproteinsD. LipoproteinsE. Glycoproteins

Proteoglycans are conjugated proteins containing different polypeptide chains of core protein and glucose aminoglycan moiety. Choose the class of conjugated protein that is related to proteoglycan:

A. PhosphoproteinB. NucleoproteinC. ChromoproteinD. GlycoproteinE. Lipoprotein

The electrophoresis method is used for the separation of blood plasma proteins. Name the parameter of the protein molecule that is in need to determine the pH of buffer solution for separation of proteins in electrophoresis method:

A. The mass of the moleculeB. The solubility in lipidsC. The diameter of the moleculeD. The solubility in the waterE. Isoelectric point

What compound serves as non-protein part of glycoproteins:

A. PhospholipidB. GalactoseC. Ferric ionD. Copper ionE. Heme

The phosphorylation (the attraction of phosphate group to the substrate) of polypeptide chain is often used for stimulation of biological activity of a protein. Name the class of conjugated protein formed due to phosphorylation:

A. NucleoproteinsB. GlycoproteinsC. PhosphoproteinsD. LipoproteinsE. Chromoproteins

Ultracentrifugation is in need in biochemical investigations for:

A. The determination of primary structure of proteinsB. The separation of low-molecular compoundsC. The study of three-dimensional structure of the moleculeD. The receiving of subcellular fractions of a cellE. The investigation of chemical composition of organic compound

All the proteins are divided into simple and conjugated ones. Find out the conjugated protein among these ones:

A. HistoneB. Egg albuminC. ProtamineD. Egg globulin

E. Myoglobin

Complex proteins do various functions. Find out the function of hemoglobin in erythrocytes:

A. Protection against virusesB. Transport functionC. Regulatory functionD. Nutrition functionE. Catalytic function

Which group of proteins being phosphoproteins posses an activity but being dephosphorylated has lost the activity:

A. Carriers through membraneB. HormonesC. EnzymesD. Transfer of lipidsE. Transfer of vitamins

Which method is better suited to separate a mixture of compounds into its individual components and detects small amounts (microgram or even picogram) of material:

A. Salting outB. DialysisC. UltracentrifugationD. SpectrophotometryE. Paper chromatography

Ceruloplasmin (the protein of blood plasma) contains copper ion and therefore has blue color. Name the class of this protein:

A. GlycoproteinsB. ChromoproteinsC. NucleoproteinD. PhosphoproteinsE. Lipoproteins

Choose, please, the blood serum index used for estimation of hyperproteinemia state:

A. The total activity of all the enzymesB. The content of albuminsC. The amino acid concentrationD. The content of acute phase proteinsE. The total content of proteins

The conjugated protein necessarily contains special component as a non-protein part. Choose the substance that can't carry out this function:

A. Salt of Hg[sup]2+</sup>B. AMPC. Thiamine pyrophosphateD. ATPE. Glucose

The conjugated protein necessarily contains special component as a non-protein part. Choose the substance that can't carry out this function:

A. Phosphate anionB. HemeC. MonosaccharideD. HNO<sub>3</sub>E. Fe[sup]2+</sup>

Point out a possible cause of hyperproteinemia

A. Diabetes mellitusB. Increased permeability of the capillary wallC. Infection (disturbed the macrophage system)D. Nephritic syndromeE. Affected gastrointestinal tract

The hormone receptors are related to the class of conjugated proteins. Name it:

A. ChromoproteinsB. GlycoproteinsC. FlavoproteinsD. LipoproteinsE. Nucleoproteins

The catalytic site of active center of enzyme is used for:

A. Binding with the substrateB. Removal of a product of the reactionC. Binding with activatorD. Conversion of a substrate in the reactionE. Binding with inhibitor

Oxidoreductase can contain prosthetic group with vitamin B<sub>2</sub>. Name it:

A. RetinalB. Ascorbic acidC. Flavin adenine dinucleotide (FAD)D. Pyridoxal phosphateE. Nicotinamide adenine dinucleotide (NAD<sup>+</sup>)

In case of enterobiasis acrihine - the structural analogue of vitamin B<sub>2</sub> - is administered. The synthesis disorder of which enzymes does this medicine cause in microorganisms?

A. AminotransferasesB. FAD-dependent dehydrogenasesC. PeptidasesD. CytochromeoxidasesE. NAD-dependet dehydrogenases

The enzyme hexokinase can catalyze the conversion of glucose or fructose in tissues. Find out the type of this enzyme specificity:

A. Relative groupB. AbsoluteC. Absolute groupD. Absolute relativeE. Stereochemical

One of the important properties of enzymes is their specificity of action. Check up a type of specificity for salivary amylase:

A. Absolute relativeB. Absolute groupC. AbsoluteD. Relative groupE. Stereochemical

Some terms are used for the description of non-protein part of an enzyme. Point out the term for non-protein part that easily dissociates from polypeptide chain:

A. Apoenzyme

B. Metal ionsC. CofactorD. Prosthetic groupE. Coenzyme

LDH1 and LDH2 levels are raised in the following organ damage:

A. Brain, bones, liverB. Liver, brain, kidneyC. Heart, RBCD. Brain, heart, liverE. Heart, kidney, liver

The oxidation of a substrate may be catalyzed by enzyme - flavoprotein that contains FAD as prosthetic group. Name, please, the vitamin used for this non-protein part of enzyme formation:

A. Ascorbic acidB. NicotinamideC. BiotinD. RiboflavinE. Adenosine triphosphate

Coenzyme forms are correctly matched to vitamins except one. Point out it:

A. Niacin – NAD<sup>+</sup> + NADP<sup>+</sup>B. Pantothenic acid – CoASHC. Vitamin B<sub>1</sub> - ATPD. Vitamin B2 – FMN + FADE. Biotin – carboxylated biotin

There are different cofactors in the structure of conjugated enzymes but only one is used for transfer of amine group from amino acid to keto acid. Name it:

A. CarboxybiotinB. Thiamine pyrophosphateC. Pyridoxal phosphateD. NAD<sup>+</sup>E. FAD

Some terms are used for the description enzyme components. Point out the term of a protein part of conjugated enzyme:

A. Metal ionsB. CofactorC. ApoenzymeD. Prosthetic groupE. Coenzyme

Which of the following is the reaction that is catalyzed by lyase:

A. OxidationB. Oxidative decarboxylationC. AcetylationD. DehydrationE. Hydrolysis

ATP molecules may be used for Transferases and Ligases function. Point out the signs of ATP use for Ligases class:

A. ATP is used for hydrolysis of a substrate bondB. ATP is used for a substrate decarboxylation.C. ATP is used for the new bond formation during the interaction of two substratesD. ATP is used for a substrate dephosphorylation

E. ATP is used for a substrate phosphorylation

Only one factor can influence on the charge of amino acid radicals in the enzyme active centre. Name it:

A. pH mediumB. The presence of a competitive inhibitorC. TemperatureD. The surplus of a reaction productE. Pressure

Researchers isolated five isozymes of Lactate dehydrogenase from human blood serum and studied their properties. What property of these isozymes indicates that they are genetic forms of the same enzyme?

A. The same electrophoretic mobilityB. The same net charge of the moleculeC. The same tissue localizationD. They have the same molecular weightE. They catalyze the same reaction

Enzymes mediating transfer of a structural fragment from one molecule to another are:

A. LigasesB. OxidasesC. LyasesD. TransferasesE. Peptidases

Point out the factor that can cause the damage of enzyme function in a cell:

A. The surplus of protons in a cellB. The presence of a product of enzymatic reactionC. pH medium about 7.2D. Temperature 37<sup>o</sup>CE. The presence of activator of enzyme

Chymotrypsin is the proteolytic enzyme catalyzing the cleavage of peptide bonds in any protein molecule. Name the class of this enzyme:

A. IsomeraseB. OxidoreductaseC. HydrolaseD. LyaseE. Ligase

The relative group specificity may be found for enzymes catalyzing the digestion of proteins in the gastrointestinal tract. Find out their trivial name:

A. TransaminaseB. Protein kinaseC. UreaseD. Protein phosphataseE. Peptidase

The change of the temperature of environment from 0<sup>o</sup>C to 38<sup>o</sup>C can cause this effect on enzyme molecule:

A. A denaturation of enzymes occursB. Enzyme action specificity variesC. The enzyme molecular charge changesD. The substrate molecular charge changesE. The probability of enzyme-substrate complex formation is increasedF.

In clinical practice tuberculosis is treated with izoniazid preparation – that is an anti-vitamin able to penetrate into the tuberculosis bacillus. Tuberculostatic effect is induced by the interference with replication processes and oxidation-reduction reactions due to the buildup of pseudo-coenzyme:

A. CoQB. TDPC. NADD. FADE. FMN

A qualitative composition of product molecule is completely identical to substrate one, but the structure is different. Name the enzyme class:

A. HydrolaseB. IsomeraseC. LyaseD. LigaseE. Oxidoreductase

Name, please, the principle base used for the classification of enzymes:

A. Chemical structure of enzymeB. Type of chemical reaction catalyzed by enzymeC. Type of energy conversionD. Chemical structure of non-protein part of the enzymeE. Chemical structure of products for enzymatic reaction

Name the vitamins whose derivatives are used for the formation of oxidoreductase structure:

A. H, DB. C, PC. B<sub>6</sub>, B<sub>9</sub>D. B<sub>2</sub>, B<sub>3</sub>E. A, E

There are some factors influencing human enzyme activity. Point out one of them resulting in complete loss of enzymatic activity:

A. Vitamin HB. Sodium chloride solutionC. P =101325 PaD. Сarbon dioxideE. T = 100<sup>o</sup> C

There are some characteristic sites in the enzyme structure. Choose the most important site for enzyme function:

A. ApoenzymeB. Active centreC. Allosteric centreD. CofactorE. A site containing Asp and Glu

Enzymes are the catalysts of protein nature. Name the property of enzymes which is not represented at the inorganic catalysts:

A. Big half-lifeB. Ability to lowering the energy to activate the reactionC. Inert to chemical substratesD. Ability to the denaturationE. Wide specificity

Which bond is cleaved by Alpha amylase in oral cavity?

A. Beta 1-6 glycosidicB. Beta 1-4 glycosidicC. Ester bond in any ester structureD. Alpha 1-4 glycosidicE. Alpha 1-6 glycosidic

The pancreatic amylase is in need to cleave the alpha-1.4-glycosidic bonds in the structure of polysaccharides using the water as the substrate. Specify the class of this enzyme:

A. LyaseB. IsomeraseC. HydrolaseD. LigaseE. Oxido reductase

Conjugated enzymes contain cofactors in their structure. Point out the location of vitamin derivative cofactor in the structure of enzyme:

A. Allosteric centreB. Near the metal-ion-cofactor in the structureC. Hydrophobic fragment of structureD. Hydrophilic fragment of structureE. Active centre

The active centre of simple enzyme is composed from:

A. Linear fragment of polypeptide chainB. The cofactor and prosthetic groupC. The terminal amino acid residuesD. One polypeptide chain completelyE. Some amino acid residues of polypeptide chain placed in the same spatial fragment

“All biological catalysts are not proteins” This statement is justified by this notion:

A. Antibodies take part in the catalysis of many reactionsB. Metal ions are involved in attachment to enzymesC. RNAs can act as ribozymesD. All enzymes do not follow the Michaelis-Menten hypothesisE. Enzyme activity may be controlled by hormones

Covalent catalysis as mechanism of enzyme catalysis was studied in experimental works with one proteolytic enzyme. Find out it:

A. AldolaseB. DecarboxylaseC. ChymotrypsinD. PepsinE. Glucoisomerase

Competitive inhibitor always interacts with enzyme active centre. Find out the explanation of this phenomenon:

A. Inhibitor is similar to the product's structureB. Inhibitor causes the denaturation of active centreC. Inhibitor is similar to a substrate structureD. Inhibitor forms a covalent type of bonds with amino acid residues of active centreE. Inhibitor is an exact copy of a substrate structure

The regulation of the enzymatic activity is carried out by different ways. Point out the way that is used more often in the regulation of key enzymes:

A. The change of pH mediumB. Competitive inhibitionC. Activation by Ca<sup>2+</sup>D. Limited proteolysisE. Allosteric regulation

Choose the name of scientists whose experiments are recognized as the basis in understanding of “induced fit” theory for mechanism of enzymatic reaction:

A. Fisher E.B. Koshland D.C. Haldane R.D. Menten M.E. Michaelis L.

Disopropyl phosphofluoride (DFP) reacts with serine proteases irreversibly and therefore is:

A. Affinity labelB. Allosteric inhibitorC. Non-competitive inhibitorD. Competitive inhibitorE. A stimulator

Succinate dehydrogenase catalyses the dehydrogenation of succinate. Malonic acid HOOC-CH2-COOH is used to interrupt the action of this enzyme. Choose the inhibition type:

A. Limited proteolysisB. AllostericC. DephosphorylationD. CompetitiveE. Non-competitive

Choose the method that is more often used for the determination of inhibitor type:

A. Michaelis-Menten graphical methodB. Nuclear magnetic resonance methodC. Eadie-Hofstee graphical methodD. Lineweaver-Burk graphical methodE. Briggs-Haldane graphical method

Covalent modification of inactive form of enzyme may be catalyzed by special enzyme in a cell. Name it:

A. HydroxylaseB. Protein kinaseC. EsteraseD. LigaseE. Oxygenase

Cholesterol synthesis is regulated by feed-back mechanism. Name the allosteric inhibitor of key enzyme for this synthesis:

A. GlucoseB. NADPHC. ADPD. CholesterolE. ATP

Glycogen phosphorylase b is transformed to the active form a by the action of special kinase with the use of ATP as donor of phosphate group. Find out, please, the type of enzyme activation:

A. Competitive inhibitionB. Activation by Ca<sup>2+</sup>C. Limited proteolysis

D. The change of pH mediumE. Covalent modification

The affinity of enzyme molecule to a substrate may be estimated using the value of:

A. pH of the environmentB. The temperatureC. V<sub>max</sub> for reaction durationD. K<sub>m</sub> for the enzymeE. The initial velocity of the reaction

The common feature of an enzyme-cascade system regulation is:

A. Suicide inhibitionB. Feed back inhibitionC. AmplificationD. Competitive inhibitionE. Counter-regulation

Point out the activator used for the determination of amylase activity:

A. H<sub>3</sub>PO<sub>4</sub>B. Ca<sup>2+</sup>C. NaClD. CuSO<sub>4</sub>E. ATP

Salivary amylase activity may be decreased by:

A. The change of pH from 6,8 to the value 5,5B. All the changes described may be in needC. The increase of temperature from 38<sup>o</sup>C to 65<sup>o</sup>CD. The decrease of temperature from 38<sup>o</sup> C to 25<sup>o</sup>CE. The addition of copper sulfate

Choose the factor that changes the cytoplasmic enzyme conformation mainly:

A. Environmental temperature value about 25<sup>o</sup> CB. Allosteric inhibitorC. Suicide inhibitorD. Environmental pH value about 7.4E. Water

A majority of key enzymes contain the allosteric centre. Specify a role of this centre:

A. It blocks the active centreB. It promotes the dissociation of a coenzymeC. It changes the structure of the substrateD. It attaches the substrateE. It attaches the regulatory factor

E. Fisher`s theory explains the mechanism of enzyme action with the fixed type of specificity, only. Name it:

A. StereochemicalB. AbsoluteC. Relative groupD. Absolute relativeE. Absolute group

Name, please, the factors that must be in constant levels during the investigation of enzyme concentration influence the velocity of enzymatic reaction:

A. Activator concentrationB. Substrate concentrationC. All the factors proposed must be in constant levelsD. The temperatureE. pH of the environment

Point out the type of bonds that are formed between Ser, Tyr and Cys residues in active centre of the enzyme and the molecule of substrate containing functional group: --COOH:

A. Ester and hydrogen bondsB. Disulfide bond and hydrophobic interactionC. Disulfide and ester bondsD. Hydrophobic interactions, onlyE. Ester bond and hydrophobic interaction

Suicide type of inhibition is considered when:

A. There is the intermediate metabolite formation from the inhibitor which tightly binds to the active centre of enzyme to block itB. The product of reaction is the allosteric inhibitor for enzymeC. The inhibitor structure is similar to substrate oneD. The end-product of reaction binds to the structure of the substrate to give non-soluble complexE. The end-product of reaction is not removed from the environment

Name, please, the equation for V-[S] dependence for the moment of complete saturation of all the active centers of enzyme molecules:

A. V=V<sub>max</sub> •[S]/K<sub>m</sub>+[S]B. V=V<sub>max</sub> / K<sub>m</sub> + [S]C. V=V<sub>max</sub>D. V=k• [S]E. V= k•V<sub>max</sub>

Find out, please, the factor used to change the charge of functional groups both in the active centre of enzyme and in the substrate molecule:

A. The addition of activator to the environmentB. The pH of environmentC. The addition of competitive inhibitor to the environmentD. The temperature of environmentE. All the factors proposed may be in need

Name, please, the inhibitor for salivary amylase:

A. Hydrogen peroxideB. Copper sulfateC. Potassium cyanideD. Sodium chlorideE. Alanine

Point out the probable way of proenzyme transformation to the active enzyme:

A. Inhibitor actionB. DecarboxylationC. Vitamin non-protein part dissociation from enzymeD. DehydrationE. Limited proteolysis

The common enzymatic reaction may be represented so: E + S <-> ES <-> ES <-> EP <-> E + P. Try to name using this equation all the factors that can influence the rate of this reaction:

A. The concentration of a substrate, enzyme and product, onlyB. The concentration of enzyme, only

C. The concentration of a substrate, enzyme, product and stability of ES-complexD. The concentration of enzyme-substrate complex, onlyE. The concentration of a substrate, only

Heme synthesis starts from glycine and succinyl-SCoA interaction with delta-aminolevulinate synthetase help. It is inhibited by the terminal metabolic product - heme. Name the inhibition type:

A. Uncompetitive InhibitionB. Limited proteolysisC. Non-competitive InhibitionD. Feedback InhibitionE. Competitive Inhibition

Sulfonamides are used as drugs to protect our organism from some foreign bacteria. Enzyme in bacterial cell producing folic acid from para-aminobenzoate is inhibited by this group of drugs. Choose the type of inhibition for sulfonamides:

A. NoncompetitiveB. UncompetitiveC. SuicideD. AllostericE. Competitive

The formation of ES complex is due to various types of bonds between E and S. Specify the type of bond, which is usually formed between charged functional groups in this case:

A. Hydrophobic interactionB. Hydrogen bondC. Peptide bondD. Donor-acceptor bondE. Electrostatic interaction

The cytoplasm fraction of tissue homogenate has been obtained for some enzyme activity determination. Point out the enzyme whose activity may be determined in this fraction:

A. PepsinB. GlucokinaseC. UreaseD. MaltaseE. Trypsin

Find out the parameter that is changed in the incubation phase of experimental test tube for the determination of choline esterase activity in the blood plasma:

A. The acetylcholine concentrationB. All the positions are rightC. The choline concentrationD. The acetic acid contentE. The pH

Twelve hours after an accute attack of retrosternal pain a patient presented a jump of aspartate aminotransferase activity in blood serum. What pathology is this deviation typical for?

A. Diabetes insipidusB. Myocardium infarctionC. Diabetes mellitusD. CollagenosisE. Viral hepatitis

Pyruvate dehydrogenase complex is multiple enzyme system because it contains:

A. Two enzymes and five coenzymesB. Three enzymes and three coenzymes

C. Five enzymes and five coenzymesD. Three enzymes and five coenzymesE. Two enzymes and one coenzyme

Total activity of enzyme depends on the:

A. pH mediumB. Enzyme concentrationC. Temperature mediumD. All the factors describedE. Substrate concentration

A patient is diagnosed with cardiac infarction. Blood test for cardio-specific enzymes activity was performed. Which of enzymes has three isoforms?

A. Creatine kinaseB. Alanine transaminaseC. Aspartate transaminaseD. Pyruvate kinaseE. Lactate dehydrogenase

During metabolic process active forms of the oxygen including superoxide anion radical are formed in the human body. With help of what enzyme is this anion inactivated?

A. PeroxidaseB. CatalaseC. Glutathione peroxidaseD. Superoxide dismutaseE. Glutathione reductase

Find out the pathological state associated with the decrease of choline esterase activity in the blood serum of patient

A. GoutB. Acute pancreatitisC. MyelomaD. InfluenzaE. Viral hepatitis

Profuse foam appeared when dentist put hydrogen peroxide on the mucous of the oral cavity. What enzyme caused such activity?

A. Glucose-6-phosphatdehydrogenaseB. AcetyltransferaseC. MethemoglobinreductaseD. CholinesteraseE. Catalase

Find out the activator used for amylase activity determination in the urine of patient:

A. PyruvateB. StarchC. Copper sulfateD. Ammonia sulfateE. Sodium chloride

Marked increase of activity of МВ-forms of CPK (creatine phosphokinase) and LDH-1 were revealed on the examination of the patient's blood. What is the most likely pathology?

A. PancreatitisB. HepatitisC. Miocardial infarctionD. Rheumatism

E. Cholecystitis

Patient’s amylase activity in the urine excesses the normal values in ten times as much. Point out the possible diagnosis:

A. AnginaB. Viral hepatitisC. InfluenzaD. Diabetes mellitusE. Sharp pancreatitis

LDH1 and LDH2 levels are raised in the following organ damage

A. Brain, heart, liverB. Heart, RBC, kidneyC. Heart, kidney, liverD. Brain, bones, liverE. Liver, brain, kidney

6 hours after the myocardial infarction a patient was found to have elevated level of lactate dehydrogenase in blood. What isozyme should be expected in this case?

A. LDH1B. LDH5C. LDH3D. LDH4E. LDH2

Point out the signs of multiple enzyme systems (MS):

A. All that is described is rightB. The MS enzymes form several different metabolic products at onceC. The MS enzymes are united by their intracellular localizationD. The MS enzymes form a single structural-functional complex

Find out the parameters that are in need to calculate specific activity of enzyme:

A. The pH value of the environment and turnover numberB. The volume of the test sample and total activityC. The product concentration in the test sample and total activityD. Turnover number and total activityE. The protein content in the test sample and total activity

A patient presents high activity of LDH1, LDH2, aspartate aminotransferase, creatine phosphokinase. In what organ (organs) is the development of a pathological process the most probable?

A. In the heart muscle (initial stage of myocardium infarction)B. In connective tissueC. In kidneys and adrenalsD. In skeletal muscles (dystrophy, atrophy)E. In liver and kidneys

Lactate dehydrogenase (LDH) isozymes catalyze the transformation of pyruvate to lactic acid in different types of tissues. Point out the structural distinctive peculiarity of each LDH isozyme:

A. Different active center structureB. Different by the quantity of subunitsC. Different by the type of coenzyme in native moleculeD. Different level of structural organization in native moleculeE. Different by the combination of subunits, forming a native molecule

The activity of choline esterase in the blood serum of patient equals 95 micromole/sec• L. Propose, please, the way for the transformation of this value into units [mmole/Lmin]:

A. Multiple this result into 1000 times, onlyB. Divide this result into 1000 times, and multiple on 60C. Divide this result into 60, onlyD. Divide this result into 1000, onlyE. Multiple this result into 60 times, only

Krebs Cycle is an amphiobolic way. Choose the explanation of this sentence:

A. The process forms NADHB. One mole of ATP is formed in one cycleC. The process forms CO[sub]2[/sub] and H[sub]2[/sub]OD. Metabolites of Krebs Cycle may be used in anabolic pathwaysE. The process is in mitochondrion

The accumulation of NADH in the matrix of mitochondria is the signal to inhibit:

A. Isocitrate dehydrogenaseB. FumaraseC. Citrate lyaseD. Malate dehydrogenaseE. Cis-Aconitase

Name, please, the enzyme from Krebs cycle catalyzing the substrate phosphorylation:

A. FumaraseB. Isocitrate dehydrogenaseC. Citrate synthaseD. Succinyl-CoA thiokinaseE. Malate dehydrogenase

Name, please, the initial substrates for Krebs cycle (first reaction):

A. Lactate and acetyl-CoAB. Oxaloacetate, onlyC. Pyruvate and oxaloacetateD. Citric acid, onlyE. Acetyl-CoA and oxaloacetate

The increase of one substrate concentration occurs the mitochondrial matrix during the inhibition of Citrate synthetase in the Krebs Cycle. Find out this substrate:

A. MalateB. Acetyl ~ SCoAC. Alpha-KetoglutarateD. GlucoseE. Fumarate

Name, please, the process that is related to the second phase of catabolic pathways in humans:

A. GluconeogenesisB. GlycolysisC. Krebs cycleD. Ammonia utilization in Urea cycleE. Proteolysis

Propose the correct continuation of the phrase: “Citric Acid Cycle is…”:

A. The main producer of reduced forms of coenzymesB. Anabolic processC. Placed in cytoplasm of a cellD. Tissue respiration phase IIIE. The main producer of energy for erythrocytes

How many moles of high energy bond containing compound are produced due to substrate phosphorylation in one round of Citric Acid Cycle:

A. OneB. FourC. TwoD. ThreeE. Twelve

Krebs cycle does not occur in:

A. LiverB. RBCC. All the positions are rightD. HeartE. Skeletal Muscle

Vitamin B[sub]1[/sub] (coenzyme TPP) is necessary for only one dehydrogenase function in Krebs Cycle. Point out it:

A. Isocitrate dehydrogenaseB. Lactate dehydrogenaseC. Malate dehydrogenaseD. Succinate dehydrogenaseE. Alpha-Ketoglutarate dehydrogenase

Exogenous substances may be involved in catabolic pathways to be used as energy sources for humans EXCEPT:

A. VitaminsB. Amino acidsC. AlcoholD. Fatty acidsE. Monosaccharides

The rate limited step for Citric Acid Cycle duration is the reaction catalyzed by:

A. Malate dehydrogenaseB. Citrate synthaseC. Isocitrate dehydrogenaseD. Alpa-ketoglutarate dehydrogenaseE. Cis-Aconitase

Find out the competitive inhibitor for succinate dehydrogenase:

A. Malic AcidB. Magnesium ionC. Citric AcidD. Fumaric acidE. Malonic acid

All these metabolic pathways or processes take place inside the mitochondria except:

A. Fatty acid beta-oxidationB. Krebs cycleC. GlycolysisD. Urea cycleE. Oxidative phosphorylation

Name, please, the key metabolite, that may be formed in catabolic pathway both for glucose and palmitic acid in aerobic condition, only:

A. PyruvateB. Acetyl-CoAC. LactateD. OxaloacetateE. Malate

Find out, please, the enzyme whose activity is inhibited mainly under the accumulation of ATP in the matrix of mitochondria:

A. Citrate synthaseB. AconitaseC. Alpha-ketoglutarate dehydrogenaseD. FumaraseE. Malate dehydrogenase

Energy production is due to catabolic pathways only. Name those one:

A. GlycogenesisB. Citric Acid CycleC. GluconeogenesisD. Fatty Acid ElongationE. Hexose Monophosphate Shunt

Choose, please, the transformation of intermediate metabolites of Krebs cycle required the function of multienzyme complex:

A. Alpha-ketoglutarate--> succinyl-CoAB. Fumarate--> malateC. Isocitrate --> alpha-ketoglutarateD. Malate --> oxaloacetateE. Citrate-->cis-aconitate

Oxidative decarboxylation reactions occur two times in Citric Acid Cycle, but the mechanism of these reactions is not the same. Choose the conversion that may be named as oxidative decarboxylation and catalyzed by multienzyme system:

A. Alpa-ketoglutarate is converted to Succinyl-CoAB. Succinate is converted to FumarateC. Isocitrate is converted to Alpa-ketoglutarateD. Citrate is converted to Cis-AconitateE. Malate is converted to Oxaloacetate

Name, please, the class of organic compound usually used as energy source for anabolic pathways in humans:

A. Nucleoside triphosphatesB. NucleosidesC. AlcoholsD. Carboxylic acidsE. Monosaccharides

Name, please, the conversion of Krebs cycle regulated by inhibitor malonic acid

A. Succinyl-CoA-->succinateB. Succinate-->fumarateC. Malate--> oxaloacetateD. Isocitrate--> alpha-ketoglutarateE. Fumarate--> malate

Name, please, the organic compound formed in Krebs cycle due to substrate phosphorylation:

A. GTPB. ATP

C. GDPD. IsocitrateE. Citrate

Two reactions of Krebs Cycle are named as oxidative decarboxylation. Point out the enzyme for this type of reaction:

A. Isocitrate dehydrogenaseB. cis-Aconitate hydrataseC. Succinate dehydrogenaseD. Succinyl~SCoA synthaseE. Citrate synthase

Magnesium and Manganese ions are in need for the function of one enzyme from this register, only. Point out it:

A. AconitaseB. FumaraseC. Succinate dehydrogenaseD. Malate dehydrogenaseE. Isocitrate dehydrogenase

Krebs cycle is regulated by the ATP/ADP ratio in aerobic cell. Point out the value of this ratio that causes the stimulation of Krebs Cycle duration:

A. _3B. _1C. _2.5D. _5E. _0.5

Name the regulatory enzyme from Citric Acid Cycle whose activity is stimulated by allosteric activator ADP at condition of its accumulation in the matrix of mitochondria:

A. Isocitrate dehydrogenaseB. Cis-AconitaseC. Alpa-ketoglutarate dehydrogenaseD. Citrate synthaseE. Succinate dehydrogenase

Isocitrate was used as an oxidized substrate in the experiment with isolated mitochondria. Specify the substance that can inhibit the isocitrate oxidative decarboxylation:

A. ATPB. GlucoseC. ADPD. CitrateE. cAMP

How many stages are considered in catabolic pathway for glucose up to the terminal products (carbon dioxide and water):

A. OneB. TwoC. FiveD. FourE. Three

Nucleoside triphosphate is formed in Krebs Cycle. Point out its abbreviation:

A. GTPB. CTPC. ATP

D. UTPE. TTP

Fluroacetate inhibits:

A. Alpha-ketoglutarate dehydrogenaseB. Malate dehydrogenaseC. Succinate dehydrogenaseD. Cis-AconitaseE. Citrate synthetase

Amphybolic process must include intermediate metabolites which are involved in both anabolic and catabolic pathways of a cell. Choose those one:

A. All the proposedB. Citric Acid CycleC. GlycolysisD. Hexose Monophosphate ShuntE. Malate-aspartate shuttle system

Only one dehydrogenase of Krebs Cycle has the non-protein part FAD. Name it:

A. Malate dehydrogenaseB. Pyruvate dehydrogenaseC. Isocitrate dehydrogenaseD. Alpha-Ketoglutarate dehydrogenaseE. Succinate dehydrogenase

Name substances which are really terminal products for catabolic pathways and for human organism:

A. Bilirubin and UreaB. ATP and Carbon dioxideC. Carbon dioxide and WaterD. Uric acid and UreaE. Amino acids and Keto acids

Name, please, the enzyme system promoting the ADP/ATP maintenance both in the matrix of mitochondria and in cytoplasma of the cell:

A. ATP synthetaseB. ATP/ADP translocaseC. Cytochrome oxidaseD. NADH-dehydrogenaseE. Succinate dehydrogenase

Name, please, the inhibitor for complex IV of respiratory chain:

A. Potassium chlorideB. Carbon monoxideC. OxygenD. Hydrogen peroxideE. Carbon dioxide

Choose, please, the P/O ratio per one mole of pyruvate involved in oxidative decarboxylation at the accumulation of 2,4-dinitrophenol in the cell:

A. ZeroB. ThreeC. FiveD. TwoE. Four

Choose, please, the number of complexes in the long respiratory chain:

A. ThreeB. TwoC. FourD. FiveE. One

Find out, please, the inhibitor for the complex III of electron transport chain in the inner membrane of mitochondria:

A. Barbituric acidB. Antimycin AC. RotenoneD. ATPE. NADH

The tissue respiration is inhibited after coal gas poisoning. Point the electron transport chain enzyme whose activity abruptly reduces in these conditions:

A. Cytochrome cB. NADH-dehydrogenaseC. Cytochrome b1D. Succinate dehydrogenaseE. Cytochrome c oxidase

Find out the property of uncoupler that helps it to penetrate across the inner membrane of mitochondria:

A. High affinity to molecular oxygenB. High solubility in waterC. High solubility in lipidsD. Very small shape of uncoupler moleculeE. All the properties proposed

Acyl-CoA dehydrogenase involved in beta-oxidation of any Fatty Acid, has the prosthetic group FAD and is placed in the inner membrane of mitochondria of cells where this process is in duration. How many moles of ATP may be synthesized due to oxidative phosphorylation per 1 mole of acyl-CoA involved in Acyl-CoA dehydrogenase reaction?

A. FourB. ThreeC. OneD. TwoE. Five

Name, please, the substrate for Cytochrome oxidase (reduced form):

A. Succinyl-CoAB. Cytochrome CC. Water moleculeD. IsocitrateE. Molecular oxygen

Which of the following inhibit complex IV of respiratory chain?

A. AntipyrineB. AmobarbitalC. CyanideD. RotenoneE. Secobarbitone

Find out the enzyme of electron transport chain that is transmembrane protein and may be considered as a pump for protons from the matrix to the intra membrane space:

A. Cytochrome CB. Succinate dehydrogenaseC. CoQD. Cytochrome C oxidaseE. Cytochrome b, only

Name the component of electron transport chain that is not related to proteins but is also acceptor of protons and electrons:

A. CoQB. Cytochrome P[sub]450[/sub]C. Cytochrome bD. Cytochrome cE. Cytochrome c oxidase

The antibiotic oligomycin has been recently used in tuberculosis treatment. Point out the process in tuberculosis bacillus that is inhibited by this drug:

A. TranslationB. Anaerobic glycolysisC. PhagocytosisD. The active transport of substances across membranesE. Oxidative phosphorylation

The isocitrate is converted into alpha-ketoglutarate in the Krebs cycle. Choose the substance that can low the P/O ratio for this reaction:

A. Isocitrate dehydrogenaseB. NAD+C. Antimycin AD. ADPE. Citrate

The electrochemical potential formation occurs on the inner membrane of mitochondria during the active work of the electron transport chain. Point the substance that can reduce the electrochemical potential value:

A. GlucoseB. 2,4-dinitrophenolC. Malonic acidD. SuccinateE. Citric acid

Name, please, the inhibitor for complex IV of electron transport chain:

A. Potassium chlorideB. OxygenC. Hydrogen peroxideD. Carbon dioxideE. Hydrogen sulfide

Find out, please, the name of the third stage of tissue respiration:

A. Oxidative phosphorylationB. Aerobic glycolysisC. The active transport of substances across membranesD. Electron transport chain functionE. Translation

Name the terminal complex of respiratory chain:

A. Cytochrome CB. NADH-dehydrogenase and Fe-S-proteins

C. Cytochrome c oxidaseD. Succinate dehydrogenaseE. Cytochrome b, Fe-S-proteins and cytochrome C1

Name, please the coefficient that can show the quantity of ATP molecules formed per one mole of oxidized organic compound due to oxidative phosphorylation:

A. P/O ratioB. The electrochemical potentialC. Protons gradientD. ADP/ATP ratioE. Respiratory control

Carbon monoxide is discussed as the suppressor of tissue respiration in muscular tissue because of its ability to be linked to:

A. All the positions are rightB. HemoglobinC. Cytochrome C oxidaseD. Myoglobin

Point out the location of the processes which take part in tissue respiration the most intensively:

A. Golgi complexB. MitochondriaC. CytoplasmD. EPRE. Lysosome

Name, please, the name of enzyme catalyzing the coupling of proton gradient use with the synthesis of ATP in mitochondria:

A. Pyruvate dehydrogenaseB. MonoaminooxidaseC. ATP synthetaseD. LipaseE. ATP/ADP-translocase

Choose, please, the value for ATP/ADP ratio at the moment when the tissue respiration has been just inhibited in the cell:

A. ATP/ADP=0.5B. ATP/ADP>1,5C. ATP/ADP=0D. ATP/ADP=0.1E. ATP/ADP=0.8

Choose, please, the P/O ratio per one mole of isocitrate involved in oxidative decarboxylation at the accumulation of rotenone in the cell:

A. 3B. ZeroC. P/O<2D. P/O<3E. 2

Protons gradient in the inner membrane of mitochondria may be increased under condition of incorporation of high doses of one vitamin in the body. Find out it:

A. Retinoic acidB. Folic acidC. CobalaminD. Pantothenic acid

E. Ascorbic acid

Choose the enzyme of Krebs cycle that is considered as the part of Complex II of electron transport chain:

A. Cytochrome oxidaseB. CoQC. NaDH-dehydrogenaseD. Succinate dehydrogenaseE. Cytochrome b, only

Name, please, the P/O ratio per one mole of pyruvate involved in oxidative decarboxylation if 1 mole of NADH is formed in this reaction:

A. FourB. ThreeC. FiveD. TwoE. Zero

Antimycin A is discussed in scientific literature as the antibiotic whose pharmacological effects are based on its ability to inhibit complex III of electron transport chain of the inner membrane of mitochondria. Choose the value for P/O ratio of succinate oxidation in Krebs Cycle at the presence of antimycin A in mitochondria:

A. _2B. _0C. Less then 3D. Less then 2E. _3

Cytochromes are:

A. Iron-porphyrin proteinsB. PeroxidasesC. Pyridine nucleotidesD. Hemoglobin derivativesE. Metal containing flavoproteins

Catalase activity is very important for cells where the accumulation of one toxic compound may be. Name, please, this compound:

A. Hydrogen peroxideB. Nitric acidC. Carbon monoxideD. Carbon dioxideE. Potassium cyanide

Name, please, the acceptor of electrons from NADH-dehydrogenase in the electron transport chain:

A. Fe-S-proteinsB. Cytochrome oxidaseC. CoQD. Cytochrome cE. Cytochrome b

Choose, please, the P/O ratio per one mole of pyruvate involved in oxidative decarboxylation at the presence of phenobarbital in high levels in the cell:

A. ZeroB. TwoC. ThreeD. Five

E. Four

Name, please, the energy effect per one Krebs cycle due to oxidative phosphorylation:

A. 9 ATPB. 11 ATPC. 12 ATPD. _1 GTPE. 8 ATP

The complex I of electron transport chain may be inhibited by barbituric acid. Find out, please, the composition of this complex:

A. NAD and Fe-S-proteinsB. FMN and Fe-S-proteinsC. Succinate dehydrogenase and Fe-S-proteinsD. NADH-dehydrogenase and Fe-S-proteinsE. FAD and Fe-S-proteins

Uncouplers of oxidative phosphorylation may be:

A. High levels of free fatty acidsB. High levels of serum bilirubinC. All the proposedD. High levels of thyroid hormonesE. Thermogenins

Rotenone (the inhibitor of the first complex of the electron transport chain) changes the P/O ratio for substrates that are oxidized in Krebs Cycle. Choose the value of P/O at the presence of this inhibitor per 1 mole of malate that is oxidized:

A. ZeroB. <4C. <2D. <3E. <1

Name, please, the factor that can block tissue respiration completely:

A. PhenobarbitalB. Malonic acidC. ValinomycinD. Potassium cyanideE. Oligomycin

What type of tissue is specialized to carry out the oxidation uncoupled from phosphorylation?:

A. Brown adipose tissueB. Skeletal muscular tissueC. White adipose tissueD. Erythrocytes of bloodE. Myocardium tissue

ATP/ADP ratio equals 0.5 in the matrix of suspended mitochondria. What enzyme systems must be stimulated in the matrix and inner membrane to increase this value up to 1?:

A. Citric Acid Cycle enzymesB. ATP-synthetaseC. Electron transport Chain enzymesD. ATP/ADP-translocaseE. All the proposed

Complete hydrolysis of 1 mole of phosphatidylcholine would yield the components: glycerol, fatty acid, phosphate and choline in which of the following respective molar ratios?

A. 1 : 2 : 1 : 1B. 2 : 1 : 1 : 1C. 1 : 1 : 2 : 1D. 1 : 2 : 2 : 1E. 1 : 2 : 1 : 2

Choose the correct continuation for the sentense: “The great majority of triacylglycerols absorbed in the intestines appears in the blood in a form of:

A. VLDLB. ChylomicronsC. HDLD. Triacylglycerols-albumin complexE. LDL

Choose the separation method for lipoproteins of the blood plasm:

A. Photocolorimetry methodB. Radioimmunal AssayC. ExtractionD. Salting-outE. Electrophoresis

Which of the following is a dietary essential?

A. GlycineB. CholesterolC. Linoleic acidD. GalactoseE. Glycerol

Which is a lipotropic factor:

A. AlanineB. CholineC. GlycerolD. InsulinE. Carnitine

Ganglioside is composed of all except:

A. Sialic acidB. GalactoseC. CeramideD. PhosphateE. Fatty acid

Free fatty acids are transported in circulation via:

A. GlobulinsB. TriacylglycerolsC. HDLD. ChylomicronsE. Albumins

Point out the terminal product of triacylglycerols lipolysis in adipose tissue:

A. Bile acidsB. 2-MonoacylglycerolC. Diacylglycerol

D. Mineral acidsE. Glycerol

The lipid which is accumulated in fatty liver is:

A. LipoproteinB. CholesterolC. TriacylglycerideD. Palmitic acidE. Sphingomyelin

Which one of the following properties is not characteristic of LDL?

A. LDL contains more cholesteryl esters than triacylglycerolsB. LDL have specific high affinity receptors in most cellsC. LDL are more dense than chylomicronsD. LDL is smaller than both VLDL and chylomicronesE. The major protein component of LDL is apoB-48

Choose the organs or tissues where lipogenesis proceeds most intensively:

A. Brain, pancreas glandB. Kidneys, adrenal glandC. MyocardiumD. Liver, Mammary glandE. Skeletal Muscle

This enzyme is present in lysosomes and specifically cleaves the bond between phosphate and glycerol of phospholipids. Point out the enzyme described above:

A. Phospholipase DB. Phospholipase BC. Phospholipase CD. Phospholipase A<sub>2</sub>E. Phospholipase A<sub>1</sub>

Which of the following statements is correct for situation when mobilization of fat from adipose tissue occurs:

A. The increase of fatty acids content in blood is observedB. Hormone-sensitive triacylglycerol lipase is dephosphorylatedC. The activity of triacylglycerol lipase is decreasedD. cAMP level is decreased in adipocytesE. The decrease of fatty acids content in blood is observed

Which of the following descriptions is not correct for hormone-sensitive triacylglycerol lipase:

A. It is synthesized in adipocytesB. It is activated by phosphorylationC. It is activated under effect of epinephrineD. It is activated under effect of glucagonE. It is activated under effect of insulin

Endogenous triglyceride carrier in plasma is mainly:

A. ChylomicronB. IDLC. VLDLD. HDLE. LDL

Point out the type of lipoproteins transporting great bulk of triacylglycerols by blood from intestine to tissues:

A. VLDLB. LDLC. ApoproteinsD. ChylomicronesE. HDL

Continue properly the proposition: “Lipoprotein lipase located in the vascular endothelium of adipose tissue…:

A. removes fatty acids from triglycerides of very low density lipoproteins (VLDL)”B. removes fatty acids from triglycerides of high density lipoproteins (VLDL)”C. is inhibited by heparin”D. hydrolyzes triglycerides to fatty acids and glycerol during lipolysis in the adipocyte”E. is increased in activity during fasting, when lipid storage is diminished”

Liver synthesizes triacylglycerols which are transported to adipose tissue by:

A. HDLB. ChylomicronesC. VLDLD. LDLE. Free fatty acids-albumin complex

Which of the following enzymes releases arachidonic acid from membrane phospholipids for its transformation into prostaglandins:

A. Phospholipase GB. Phospholipase A<sub>2</sub>C. Phospholipase DD. Phospholipase CE. Phospholipase A<sub>1</sub>

Point out the enzyme used for lysophospholipid formation during lipolysis of glycerophospholipid:

A. Diacylglycerol lipaseB. Phospholipase DC. Phospholipase A<sub>2</sub>D. Monoacylglycerol lipaseE. Triacylglycerol lipase

Alcocholic group is found in

A. PhosphatidylcholineB. PlasmalogenC. GalactosylceramideD. CephalinE. Lecithin

Which of the following phosphorylated enzymes are active?

A. Pyruvate kinaseB. Pyruvate dehydrogenase complexC. Hormone-sensetive triacylglycerol lipaseD. Acetyl CoA carboxylaseE. Glycogen synthase

Lipids are natural organic compounds that are:

A. Soluble in buffer solutionsB. Good soluble in waterC. Insoluble in benzeneD. Soluble in organic solvents

E. Insoluble in organic solvents

In man the serum lipoprotein fraction with the highest free cholesterol content is:

A. Beta-lipoproteinB. ChylomicronsC. Alpha-lipoproteinD. Pre-beta-lipoproteinE. Prealbumin

Choose the incorrect statement about the glycerol use as a substrate for gluconeogenesis:

A. Glycerol is released from adipocyte being phosphorylated beforeB. The liver takes up the glycerol and phosphorylates itC. Glycerol phosphorylation is supplied with 1 ATPD. Conversion of glycerol to glycerol-3-phosphate is catalyzed by glycerol kinase.E. Glycerol is released from adipose stores of triacylglycerol

There are two sources of glycerol 3-phosphate for triacylglycerol synthesis. Adipose tissue is strictly dependent on glucose uptake to produce dihydroxyacetone phosphate for glycerol 3-phosphate formation. But liver can use diverse (glucose independent) way for glycerol 3-phosphate synthesis. Point out the liver specific enzyme for this transformation:

A. EnolaseB. Glycerol kinaseC. Glycerol 3-phosphate dehydrogenaseD. Acetyl CoA carboxylaseE. Aldolase

Point out the most atherogenic lipoproteins of the blood plasm:

A. IDLB. HDLC. LDLD. ChylomicronsE. Alpha-lipoproteins

In electrophoresis which class of lipoproteins will least migrate from start-line:

A. LDLB. HDLC. IDLD. VLDLE. Chylomicrons

Point out the lipoprotein class transporting cholesterol from peripheral tissues to the liver:

A. ChylomicronesB. Cholesterol-albumin complexC. HDLD. LDLE. VLDL

Which of the following is an activator of LCAT (Lecithin-cholesterol acyltransferase)

A. Apo CIIB. Apo AIC. Apo B 100D. Apo EE. Apo B 48

Certain drugs - caffeine and theophylline - inhibit phosphodiesterase activity catalyzing reaction cAMP-->AMP. How will free fatty acid levels are altered in the blood after the drugs injection:

A. Will be increased simultaneously with galactoseB. Will be not alteredC. Will be decreasedD. Will be increasedE. Will be decreased simultaneously with glucose

Point out the lipoproteins of blood plasma that keep the biggest amount of cholesterol ester:

A. HDLB. VLDLC. LDLD. ChylomicronsE. IDL

All of the following statements about LDL are true except:

A. It contains apo B-100B. 25% of its content is utilized in the liverC. It delivers cholesterol to cellsD. It is a marker for cardiovascular diseaseE. It contains only one apoprotein

Which statement is incorrect about phosphatidylcholine (PC)?

A. PC participates in signal transmission via activated phospholipase C.B. PC may be synthesized from phosphatidylserine or from diacylglycerol and CDP-choline.C. PC is an important component of the surface shell of plasma lipoproteins.D. PC can be hydrolyzed by phospholipase A2 into lysolecithin and a fatty acid.E. PC together with phosphatidylethanolamine and phosphatidylserine are major phospholipid components of cell membranes.

Synthesis of phospholipids is disordered under the liver fat infiltration. Indicate which of the following substances can enhance the process of methylation during phospholipids synthesis?

A. GlycerolB. Ascorbic acidC. GlucoseD. CitrateE. Methionine

Glycerol must be activated prior to the synthesis of triacylglycerols. Point out the enzyme realizing this activation:

A. Glycerol dehydrogenaseB. Glycerol 3-phosphate acyl transferaseC. Glycerol kinaseD. Cytochrome P450E. Glycerol-CoA synthase

The lipids are transported by lipoproteins in the blood. Specify the lipoproteins that are formed in the small intestine wall after high lipids intake:

A. LDLB. VLDLC. HDLD. IDLE. Chylomicrons

If choline moiety is replaced by ethanolamine in glycerophospholipid the net product is:

A. PlasmalogenB. Sphingomyelin

C. CardiolipinD. CerebrosideE. Cephalin

Point out the biological role that is suitable for phospholipid:

A. Component of multiple enzyme systemB. Membrane structural componentC. Substrate for the bile acid formationD. Precursor for 1, 25-dihydroxycholecalciferol synthesisE. Substrate for steroidal hormones synthesis

Point out the lipoproteins of the blood plasma containing the highest mass of triacylglycerols:

A. HDLB. VLDLC. LDLD. IDLE. Chylomicrons

What substance is not component of a lipoprotein:

A. TransferrinB. ApoproteinC. PhospholipidD. TriacylglycerolE. Cholesterol

The liver requires a mechanism for producing phosphatidylcholine (PC) because it exports significant amounts of PC in the bile and as component of serum lipoproteins. This mechanism includes three methylation steps to produce PC from phosphatidylethanolamine. What is the methyl group donor for methylation:

A. Uridine diphosphate-methionine (UDP-methionine)B. N-guanosylmethionine (NGM)C. S-adenosylmethionine (SAM)D. Cytidine diphosphate-choline (CDP-choline)E. Homosycteine

Hormone-sensitive triacylglycerol lipase is activated in adipocytes under the emotional stress. Choose the substance, whose concentration is increased in adipose tissue in this state:

A. cAMPB. cGMPC. AMPD. DiacylglycerolE. Ca<sup>2+</sup>

Choose the phospholipase acting on phosphatidyl inositol to liberate arachidonic acid, the substrate for the synthesis of prostaglandins:

A. Phospholipase DB. Phospholipase CC. Phospholipase A<sub>3</sub>D. Phospholipase A<sub>2</sub>E. Phospholipase A<sub>1</sub>

The largest reserve of energy in body is stored as:

A. Blood glucoseB. Muscle proteinC. Liver glycogenD. Triacylglycerols in adipose tissue

E. Muscle glycogen

Glycerol is converted to glucose in:

A. HeartB. Skeletal muscleC. LiverD. AdipocyteE. Brain

In diabetes mellitus the factor limiting synthesis of triglycerides in adipose tissue is

A. Acetyl CoAB. ATPC. LecithinD. Alpha-glycerophosphateE. ADP

Glycerol as product of lipolysis may be transported to the liver where it is phosphorylated to glycerol 3-phosphate and may enter glycolysis due to glycerol 3-phosphate dehydrogenase to form:

A. 1,3-BiphosphoglycerateB. 3-PhosphoglycerateC. PhosphoenolpyruvateD. Glyceraldehyde 3 phosphateE. Dihydroxyacetone phosphate

Find out the enzyme of tissue lipolysis that is regulated by hormones:

A. Triacylglycerol lipaseB. Diacylglycerol lipaseC. Lipoprotein lipaseD. Lecithin cholesterol acyl transferaseE. Monoacylglycerol lipase

Name the condition in the liver cell causing the triacylglycerol synthesis:

A. The accumulation of carbon dioxideB. The stimulation of protein degradationC. The decrease of ATP/ADP ratioD. The ketone bodies accumulationE. The accumulation of high fatty acids

Vitamin F is recommended in order to prevent the cholesterol deposition inside the blood vessels at atherosclerosis. What lipid is the component of this vitamin?

A. CardiolipidB. Stearic acidC. Linoleic acidD. SphingomyelinE. Cholesterol

What are the constituents of cardiolipin are obtained as the result of its degradation?

A. 2 glycerols, 4 fatty acids, 1 phosphateB. 3 glycerols, 4 fatty acids, 2 phosphatesC. 1 sphingosine, 1 fatty acid, 1 sugarD. 2 sphingosines, 2 fatty acids, 1 oligosaccharideE. 2 glycerols, 2 fatty acids, 1 phosphate

Hormone sensitive triacylglycerol lipase is not activated by one hormone from following list. Point out it.

A. None of above

B. GlucagonC. InsulinD. EpinephrineE. Norepinephrine

Choose the enzyme participating in phosphate elimination from phosphatidic acid:

A. Phosphatidate PhosphataseB. Phospholipase А<sub>1</sub>C. Phospholipase CD. Phospholipase А<sub>2</sub>E. Phospholipase D

Blood plasma of patient with hyperlipoproteinemia type I remains milky even after a long fast due to markedly elevated and persistent chylomicrons. What abnormality is possible in these patients?

A. Deficient lecithin cholesterol acyltransferase (LCAT)B. Defective synthesis of apoB-48C. Obstruction of the bile ductD. Deficient lipoprotein lipaseE. Deficient pancreatic lipase

The increased levels of blood plasma free fatty acids is the result of mobilization of triacylglycerols from:

A. KidneyB. BrainC. Adipose tissueD. IntestineE. Skeletal muscle

Which statement is correct about high density lipoproteins (HDL)?

A. Nascent HDL pick up triacylglycerols from peripheral cellsB. Lecithin cholesterol acyltransferase is activated by apoC-II in HDLC. Lecithin cholesterol acyltransferase produces bile acids from cholesterol in HDLD. HDL transfer cholesteryl esters to liverE. HDL are synthesized in the adrenal gland

Which statement is a true for the comparison of phospholipids(PL) and triacylglycerols (TAG)?

A. Both TAG and PL are the primary storage form for fats in our bodies.B. TAG may be saturated or unsaturated, but all fatty acids in PL structure are saturatedC. PL molecules have a distinctly polar 'head' and a distinctly non-polar 'tail,' whilst triglycerides are predominantly non-polar.D. PL contein a cyclic steroid ring, whilst triglycerides maintain a straight-chain form.E. Both molecules contain a phosphate group.

Point out, how fatty acids are activated in catabolic process:

A. Are linked to HS-CoA without any energy useB. Don't change the structureC. Are converted to acyl-SCoA due to АТP energyD. Are phosphorylated by ATPE. Interact with carnitine

Fatty acid oxidation occurs in:

A. CytosolB. Golgi apparatusC. Endoplasmic reticulumD. MitochondriaE. Ribosome

Point out the cellular location of saturated HFA synthesis:

A. NucleusB. CytoplasmC. Endoplasmic reticulumD. PlasmolemmaE. Mitochondrion

Name the ketone bodies which are excreted in diabetic ketoacidosis:

A. Acetoacetic and alpha-ketoglutaric acidsB. Acetoacetic and beta-hydroxybutiric acidsC. Acetoacetic and pyruvic acidsD. Acetoacetic and pyruvic acidsE. Acetoacetic and oxaloacetic acids

In uncontrolled diabetes mellitus, acetoacetic acid and beta-hydroxybutyric acid are produced in:

A. PancreasB. Small intestineC. BrainD. KidneysE. Liver

A 1 y.o. child with symptoms of muscle affection was admitted to the hospital. Examination revealed carnitine deficiency in muscles. Biochemical base of this pathology is disturbed process of:

A. Transporting of fatty acids to the matrix of mitochondriaB. Substrate phosphorylationC. Lactic acid utilizationD. Actin and myosin synthesisE. Regulation of Ca<sup>2+</sup> level in mitochondria

The formation of the “active form” of a fatty acid is endergonic process in which the ATP energy is consumed. But there is another necessary participant of the fatty acid activation. Choose it:

A. Acetyl CoAB. UTPC. Succinyl CoAD. CoASHE. GTP

Find out the main substrates for the use by elongase system during the formation of stearyl CoA from palmitoyl CoA:

A. Glycerol, NADPH, palmitoyl CoAB. Malonyl CoA, NADPH, palmitoyl CoAC. Malonyl CoA, NADH, palmitoyl CoAD. Acetyl CoA, NADH, palmitoyl CoAE. Acetyl CoA, NADPH, palmitoyl CoA

The important catabolic processes are located in the mitochondrial matrix. Find out the catabolic process that isn’t located in the mitochondrion:

A. Oxaloacetate formation from pyruvateB. The Krebs cycleC. GlycolysisD. Oxidative decarboxylation of pyruvateE. Beta-oxidation of high fatty acids

Choose the allosteric activator of acetyl-CoA-carboxylase (the key enzyme of HFA synthesis):

A. Oxaloacetate

B. SuccinateC. FumarateD. MalateE. Citrate

An experimental animal has been given excessive amount of carbon labeled glucose for a week. In what compound can the label be found?

A. MethionineB. Vitamin AC. PhenylalanineD. Palmitic acidE. Arachidonic acid

Point out the terminal product in the last round of &beta;-oxidation of High Fatty Acids with odd number of carbon atoms:

A. Butyryl-SCoAB. Malonyl-SCoAC. Acetoacetyl-SCoAD. Propionyl-SCoAE. Pyruvate

The most important source of reducing equivalents for fatty acid synthesis in the liver is the process named:

A. GlycolysisB. The pentose phosphate pathwayC. GlycogenolysisD. Oxidation of acetyl coenzyme AE. The citric acid cycle

It is established, that the high fatty acid radical is lengthened in two carbon atoms by palmitate synthetase complex action each cycle. Point out the donor of these two carbon atoms during the synthesis:

A. Acetyl-CoAB. Stearyl-CoAC. Lauryl-CoAD. Palmityl-CoAE. Malonyl-CoA

What compound production and utilization become more significant during starvation?

A. Uric acidB. Fatty acidsC. Ketone bodiesD. GlycogenE. Triacylglycerols

Which of the following substances is immediate precursor of acetoacetate in pathway ketogenesis?

A. Beta-hydroxy-beta-methylglutaryl CoAB. Beta-hydroxybutyryl CoAC. Acetoacetyl CoAD. Acetyl CoAE. Beta-hydroxybutyrate

There is a tissue hypoxia at myocardial ischemia in the patient. Name the process of lipid metabolism, whose rate is reduced in the myocardium at this state:

A. Phospholipid synthesisB. Cardiolipin synthesisC. Beta-oxidation of high fatty acids

D. Ketone body synthesisE. Fat lipolysis

What sentence is reasonable in regard to acetyl CoA:

A. It cannot be formed from proteinB. It serves as a donor of acetyl groups in fatty acid synthesis.C. It is carboxylated by pyruvate carboxylase to form malonyl CoAD. It cannot be used to make cholesterol and ketone bodiesE. It allosterically activates pyruvate dehydrogenase

Which of the following statements describes correctly ketone bodies?

A. They are accumulated at diabetes mellitus after insulin therapyB. They are most important nutrients for liverC. They are accumulated in children with fatty acid oxidation disordersD. They are produced by muscle but not by liverE. They include &beta;-hydroxybutyrate, acetoacetate and acetone

It is established, that &beta;-oxidation of high fatty acids is carried out by multienzyme complex in cells. Choose the enzyme that is not the component of this complex:

A. ThiolaseB. 3-Hydroxy-acyl-CoA-dehydrogenaseC. Acyl-CoA-dehydrogenaseD. Enoyl – CoA - hydrataseE. Aldolase

Which of the following enzyme controls a committed step in fatty acid synthesis:

A. ThioesteraseB. Acetyl transacylaseC. Ketoacyl synthaseD. Malonyl transacylaseE. Acetyl CoA carboxylase

Acetyl CoA is the source of carbon atoms while NADPH provides the reducing equivalents for fatty acid synthesis. Which of the following enzymes take part in the formation of both mentioned substrates (Acetyl CoAandNADPH):

A. Glucose 6 phospate dehydrogenase (G 6-P DH), gluconolactone hydrolase, phosphogluconate dehydrogenaseB. Citrate lyase, G 6-P DH, malic enzymeC. Pyruvate carboxylase, PDH, acylCoA dehydrogenaseD. Citrate synthase, pyruvate dehydrogenase (PDH), Glucose 6 phospate dehydrogenase (G 6-P DH)E. Citrate lyase, PDH, pyruvate kinase

Point out the process or reaction where acetone is formed as end- product:

A. Decarboxylation of beta-hydroxybutyric acidB. Synthesis of HFAC. Beta-oxidation of HFAD. Condensation of two acetyl-CoA moleculesE. Decarboxylation of acetoacetic acid

Point out the intracellular location of high fatty acid beta-oxidation:

A. NucleusB. LysosomeC. Outer mitochondrial membraneD. Inner mitochondrial membraneE. Mitochondrial matrix

Which of the following steps is involved in the formation of glucose from lipolysis product?

A. Glycerol from lipolysis is converted to triacylglycerolsB. Fatty acids from lipolysis are oxidized, producing FADH<sub>2</sub> and stimulating gluconeogenesisC. Fatty acids from lipolysis are converted to glucoseD. 2 glycerols from lipolysis are phosphorylated, converted in a few steps to fructose-1,6-bisphosphate, and eventually converted to glucoseE. 2 glycerols from lipolysis are taken up by liver cells and dimerized to fructose

Point out the terminal product of beta-oxidation of Higher Fatty Acids (HFA) with even number of carbon atoms:

A. Propionyl-SCoAB. Malonyl-SCoAC. Acetyl-SCoAD. Butiryl-SCoAE. Acetoacetyl-SCoA

Point out the ketone body that is not utilized in human organism:

A. Beta-hydroxybutyrateB. AcetoneC. None of the substances stated aboveD. AcetoacetateE. All the substances stated above

The most important source of reducing equivalents (NADPH) for fatty acid synthesis in the liver is:

A. Oxidation of glucuronic acidB. GlycolysisC. The citric acid cycleD. The pentose phosphate pathwayE. Oxidation of acetyl CoA

A sportsman was recommended to take a medication that contains carnitine in order to improve his results. What process is activated by carnitine the most?

A. Synthesis of ketone bodiesB. Synthesis of proteinsC. Synthesis of steroid hormonesD. Fatty acids transport to mitochondriaE. Tissue respiration

Ketone bodies are normally synthesized from:

A. Acetyl-CoAB. GlucoseC. AcetoneD. Propionyl-CoAE. Glycerol

The energy yield by stearic acid oxidation is:

A. 12B. 96C. 146D. 129E. 38

A microsomal enzyme system is responsible for the formation of some unsaturated fatty acids. Point out an enzyme of the system:

A. Cytochrome oxidaseB. Succinate coenzyme Q reductaseC. NADH coenzyme Q reductaseD. Coenzyme Q-cytochrome c reductaseE. NADH-cytochrome b5 reductase

How many ATP molecules are formed in a case of stearic acid beta-oxidation:

A. 56B. 129C. 147D. 18E. 7

Humans cannot achieve a NET synthesis of glucose from C-even fatty acids due to the inability to convert:

A. Acetyl-CoA to pyruvateB. Methylmalonyl-CoA to succinyl-CoAC. Acetyl-CoA to malonyl CoAD. Acetyl-CoA to acetoacetateE. Oxaloacetate to pyruvate

Choose the organs and tissues where lipogenesis occurs most intensively:

A. LiverB. MyocardiumC. MuscleD. KidneysE. Brain

The acetyl CoA required for fatty acid synthesis is formed in the cytoplasm as a result of one enzyme activity. Point out this enzyme:

A. Malic enzymeB. Cytrate lyaseC. Isocitrate dehydrogenaseD. ThiolaseE. Pyruvate decarboxylase

Beta-oxidation of odd-chain fatty acids produces:

A. Acetoacetyl-CoAB. PyruvateC. Propionyl-CoAD. Malonyl-CoAE. Geranyl-CoA

Point out the intracellular location of high fatty acid synthesis:

A. NucleusB. Outer mitochondrial membraneC. Mitochondrial matrixD. Inner mitochondrial membraneE. Cytosol

The removal of two-carbon units from a fatty acyl CoA involves four sequential reactions. Which of the following reaction sequences is correct for the pathway of &beta;-oxidation:

A. Reduction, dehydration, reduction, cleavageB. Reduction, hydration, dehydrogenation, cleavageC. Dehydrogenation, hydration, dehydrogenation, cleavageD. Oxidation, dehydration, oxidation, cleavage

E. Hydrogenation, dehydration, hydrogenation, cleavage

All of the following statements about ketone bodies are true except one. Point out it:

A. Acetoacetate is reduced to &beta;-hydroxybutyrateB. Acetoacetate is decarboxylated to acetoneC. Ketone bodies are formed and utilized in the liverD. The increased glucagon/insulin ratio promotes ketone bodies synthesis in patient with diabetes mellitusE. Ketone bodies are formed in the liver but utilized in the extrahepatic tissues

This vitamin is precursor for the formation of non-protein parts of both enzymes: acyl CoA synthase and fatty synthase complex. Name it:

A. ThiamineB. Pantothenic acidC. BiotinD. RiboflavinE. Nicotinic acid

There are two end products in the &beta;-oxidation of odd chain fatty acids. They are:

A. Acetyl CoA and butiryl CoAB. Acetyl CoA and propionyl CoAC. Acetyl CoA and oxaloacetateD. Methylmalonyl CoA and malonyl CoAE. Acetyl CoA and malonyl CoA

A microsomal enzyme system called fatty acyl CoA desaturase is responsible for the formation of monounsaturated fatty acids: oleic and palmitoleic acids. Name the coenzyme which is an electron donor for the enzyme system:

A. CoASHB. BiotinC. PhosphopantetheineD. NADHE. TPP

Acetyl CoA carboxylase is key enzyme in fatty acid synthesis. Point out the coenzyme of this enzyme:

A. BiotinB. PhosphopantetheineC. NADHD. FADH<sub>2</sub>E. CoASH

Point out the metabolite, concerning to ketone bodies:

A. Beta-hydroxybutyric acidB. Palmitoleic acidC. Malonyl - SCoAD. Butyric acidE. Acetyl - SCoA

Which of the following organs (tissues) cannot use ketone bodies:

A. BrainB. LiverC. Skeletal muscleD. KidneyE. Myocardium

Point out the biological role of carnitine in cells:

A. AntioxidantB. Allosteric activator of enzymesC. The component of respiratory chainD. Transporter of fatty acid across the mitochondrial membranesE. The enzyme inhibitor

Point out the substrate for acyl-CoA-dehydrogenase (beta-oxidation of HFA):

A. Butyryl-SCoAB. Beta-hydroxyacyl-SCoAC. Enoyl-SCoAD. Acetyl-SCoAE. Beta-ketoacyl-SCoA

Choose the products for one round of stearic acid &beta;-oxidation:

A. 1 Oleyl CoA, 12 ATPB. 129 ATPC. 1 palmitoyl CoA, 1 acetyl CoA, 1 FADH<sub>2</sub>, 1 NADHD. 2 acetyl CoA, 2 FADH<sub>2</sub>, 1 ATPE. 1 stearyl CoA, 1 acetyl CoA, 1 FADH<sub>2</sub>, 1 NADH

Acetyl-CoA carboxylase requires for maximal activity all the compounds listed below except one. Point out it:

A. Carbon dioxideB. AMPC. BiotinD. CitrateE. ATP

What sentence is reasonable in regard to fatty acid synthesis:

A. It occurs in the mitochondrial matrixB. It requires specific enzyme propionyl Co A carboxylaseC. It requires NADPH and ATPD. It is catalysed by fatty acid synthase, an enzyme complex that requires biotin as a coenzymeE. It is activated by glucagons

Choose the incorrect statement related to biological role of cholesterol:

A. It is precursor for bile saltsB. It is precursor for glucocorticoidsC. It is an energy source for cellsD. It is precursor for sex hormonesE. It is a structural component of membrane

Find out the product of cholesterol transformation in the liver whose content is important for lipids digestion duration in the small intestine:

A. Glycocholic acidB. TaurineC. Acetic acidD. Butyric acidE. Acetone

Which is not seen in cholesterol synthesis:

A. PyruvateB. Geranyl pyrophosphateC. SqualeneD. Lanosterol

E. Mevalonate

Find out the substance synthesized from cholesterol in human organism:

A. AldosteroneB. CalcitriolC. Lipoic acidD. CortisolE. The positions A, B, C above are right

What plasma lipoproteins, whose levels rise, are inversely correlated with atherosclerosis? :

A. Fatty acid-albumins complexB. Remnant chylomicronesC. High density lipoproteinsD. Low density lipoproteinsE. Nascent chylomicrones

Which of the following is found in conjugation with bile acids:

A. Hydrochloric acidB. Cholyl acetyl-CoAC. Cholic acidD. PregnenoloneE. Glycine

Point out the hormone that decreases the rate of lipolysis in adipose tissue:

A. InsulinB. EpinephrineC. SomatotropinD. GlucagonE. ACTH

Squalene is the intermediate product during synthesis of:

A. Arachidonic acidB. HaemC. AlanineD. CholesterolE. Palmitic acid

Find out the enzyme system that is used for bile acids formation from cholesterol:

A. 7-alpha hydroxylaseB. Acetoacetyl-CoA reductaseC. Acetyl~SCoA carboxylaseD. Beta-hydroxybutyryl dehydrogenaseE. Malonyl ~ SCoA-ACP transferase

The synthesis of 1,25-dihydroxycholecalciferol take place:

A. In the kidney from 25-hydroxycholecalciferolB. It is not synthesized in mammalsC. In the intestine from cholecalciferolD. In the liver from cholecalciferolE. In the skin under action of ultraviolet light from 7-dehydrocholesterol

Examination of cell culture got from a patient with lysosomal pathology revealed accumulation of great quantity of lipids in the lysosomes. What of the following diseases is this disturbance typical for?

A. GalactosemiaB. Gout

C. Wilson diseaseD. PhenylketonuriaE. Tay-Sachs disease

Name lipoproteins whose content is in need to determine in the blood plasma of patient with atherosclerosis of blood vessels:

A. VLDLB. ChylomicronsC. HDLD. The positions A, B, C above are rightE. LDL

Point out the atherogenic lipoproteins:

A. Low density lipoproteinsB. Remnant chylomicronesC. Fatty acid-albumins complexD. High density lipoproteinsE. Nascent chylomicrones

Which of the following statements is believed to be the most important step in the regulation of the rate of cholesterol biosynthesis from acetyl-CoA?

A. The reactions involved in the cyclization of the polyisoprenoid precursor of the sterol ring systemB. The enzymatic reduction of HMG-CoA.C. The condensation of malonyl-CoA with acetyl-CoAD. The cleavage of HMG-CoA to form acetoacetate and acetyl-CoAE. The introduction of the 3-beta-hydroxyl group into the steroid ring system.

Rate limiting step in cholesterol synthesis is:

A. HMG CoA reductaseB. Mevalonate synthaseC. HMG CoA synthetaseD. AcylCoA synthaseE. HMG CoA lyase

Find out pathological state associated with hypercholesterolemia in patient:

A. Position B, C, D are rightB. Non-insulin dependent diabetes mellitusC. CholestasisD. Insulin-dependent diabetes mellitusE. Nephritic syndrome

What stage is not from cholesterol synthesis?

A. Synthesis of squaleneB. Production of isoprenoid unitsC. Synthesis of beta-hydroxy beta-methylglytaryl CoA (HMG CoA)D. Formation of mevalonateE. Formation of acetoacetate

Gaucher`s disease is due to the accumulation of:

A. GlucocerebrosideB. TriacylglycerolC. GalactoseD. SphingomyelinE. Glucose

Point out the drug used for the decrease of cholesterol level in the blood of patients - allosteric inhibitor for key enzyme of cholesterol synthesis:

A. IndomethacinB. AspirinC. Antimicin AD. LovastatinE. Barbiturate

The biosynthesis of triacylglycerols in adipose tissue cells requires:

A. Increased concentration of fatty acids in the cellsB. Decreased levels of plasma insulinC. Increased intracellular levels of cAMPD. Elevated levels of plasma epinephrineE. Increased rate of glycerol release from the cells

The analysis of the blood of the patient has revealed the considerable growth of VLDL and autoimmune LDL levels. Specify the pathology that can be in the patient:

A. AcromegalyB. PhenylketonuriaC. Diabetes insipidusD. Viral hepatitisE. Atherosclerosis

The main chemical compounds for cholesterol synthesis are:

A. Acyl CoA, carnitine, ATPB. Acetyl CoA, GTP, pantothenateC. Malonyl CoA, NADPH, ATPD. Acetyl CoA, NADPH, ATPE. Glycerol, phosphate, choline

The enzyme system for cholesterol biosynthesis is located in the:

A. Golgi apparatusB. Plasma membraneC. Cytosol and EPR (smooth part)D. Cytosol and mitochodrial membraneE. Mitochodrial matrix

Point out the organ or tissue where the rate of cholesterol synthesis is least of all:

A. GonadsB. IntestineC. Adrenal glandD. SkinE. Kidney

All hereinafter stated compounds are products of cholesterol catabolic conversion except one. Choose it:

A. Cholic acidB. GlucocorticoidsC. Glucuronic acidD. Chenodeoxycholic acidE. Mineralocorticoids

In Niemann-Pick`s disease the following substances accumulate in CNS in excess levels:

A. LysophosphingosidesB. Cholesterol estersC. Phosphoinositides

D. SphingomyelinsE. Triacylglycerols

Point out the key regulatory enzyme of cholesterol biosynthesis in the liver:

A. Mitochondrial beta-hydroxy beta-methylglytaryl CoA (HMG CoA) synthaseB. ThiolaseC. HMG CoA reductaseD. Mevalonate kinaseE. Cytosomal HMG CoA synthase

The conversion of cholesterol catalyzed by 7&alpha;-hydroxylase is inhibited by:

A. Glycine and taurineB. CalcitriolC. CortisolD. TestosteroneE. Bile acids

Choose the number of Ch (LDL) + Ch(VLDL)/ Ch(HDL) ratio which is peculiar to healthy people:

A. 0.5-1.0B. 2.0-3.5C. 1.0-2.0D. 4.0-6.0E. 7.0-8.0

Choose the substance, whose level is increased in the blood serum of patient with atherosclerosis of blood vessels:

A. CholesterolB. High fatty acidsC. CarnitineD. HemoglobinE. Albumins

One of the following is scavenger of cholesterol from tissue and prevents atherosclerosis:

A. HDLB. LDLC. ChylomicronsD. VLDLE. Carnitine

Which of the following statements is not true concerning control of plasma cholesterol levels?

A. Drugs such as lovastatin decrease blood cholesterol levels due to inhibition of beta-hydroxy beta-methylglytaryl CoA (HMG CoA) reductaseB. Dietary intake of polyunsaturated fatty acids reduces the plasma cholesterol levelsC. Decreased levels of cholesterol is observed in people with obesity, atherosclerosis, high low density lipoproteins (LDL) levelsD. Dietary intake of large amount of sucrose and fructose increases the plasma cholesterol levelsE. Certain plant sterols reduce plasma cholesterol levels

All of the following statements regarding hypercholesterolemia (type IIa hyperlipidemia) are correct except:

A. The blood serum triacylglycerol levels are elevatedB. The blood serum low density lipoprotein (LDL) levels are highC. It is due to a deficiency of LDL receptorsD. The blood serum cholesterol levels are increasedE. There is an increased risk of coronary artery disease

The best marker for dislipidemia is:

A. Blood cholesterolB. Serum cholesterolC. Cholesterol/HDLD. LDL/HDLE. Cholesterol/TG

The rate limiting enzyme in bile acid synthesis is:

A. 7-Alpha hydroxylaseB. HMG-CoA synthetaseC. DecarboxylaseD. HMG-Co-A reductaseE. 7-Beta hydroxylase

Which of the following statements explains correctly metabolic alterations that are specific for persons disposed to obesity beside people having standart weight?

A. Rate of Krebs cycle reactions is higherB. Calorie intake is much lessC. There is any genetic defect in leptinD. Coupling of respiration with oxidative phosphorylation is much moreE. Rate of fatty acid &beta;-oxidation is much less

Cholesterol is the precursor for synthesis of all pointed below hormones except:

A. ProgesteroneB. CortisolC. InsulinD. TestosteroneE. Aldosterone

The patient with diabetes mellitus has been delivered in hospital in the state of unconsciousness. Arterial pressure is low. The patient has acidosis. Point substances, which accumulation in the blood results in these manifestations:

A. AminesB. Cholesterol estersC. Ketone bodiesD. High fatty acidsE. Monosaccharides

Point out the substance that decreases the rate of cholesterol synthesis:

A. ThyroxinB. GlucoseC. PhosphateD. CholesterolE. Insulin

Point out the substrate of cholesterol synthesis:

A. LactateB. Acetic acidC. Oxaloacetic acidD. Acetyl CoAE. Pyruvate

Which one of the following statements regarding bile salt is correct?

A. It acts in emulsification of fats in alkaline mediumB. It acts in emulsification of fats in acidic medium

C. Helps in absorption of water soluble vitaminsD. Helps in absorption of proteinsE. Helps in absorption of carbohydrates

Point out the key enzyme of cholesterol synthesis:

A. Palmitate synthetaseB. Acetyl~SCoA carboxylaseC. Beta-hydroxybutyryl dehydrogenaseD. Beta-hydroxy-beta-methylglutaryl-CoA reductaseE. Malonyl~SCoA-ACP (Acyl Carrier Protein) transferase

The liver is major site of cholesterol biosynthesis, although other tissues are also active in this regard.The terminal products of cholesterol catabolism in the liver play the important role during lipids intake in human organism Point out these products:

A. Bile acidsB. CorticosteroidsC. CatecholaminesD. Cholanic acidE. Acetyl - SCoA

Find out the coenzyme used in some reactions of cholesterol synthesis:

A. BiotinB. NADHC. NADPHD. FADE. Pyridoxal phosphate

Which statement is not associated with situation of fat deposition?

A. Blood free fatty acid levels are elevatedB. Blood VLDL and chilomicrone levels are elevatedC. Lipoprotein lipase activity is increasedD. Insulin secretion is increasedE. Hormone sensitive triacylglycerol lipase activity is increased

Which of the following statements is not true for fat mobilization description?

A. Blood epinephrine levels are elevatedB. Blood glucagon levels are elevatedC. Hormone-sensitive lipase is phosphorylatedD. Hormone-sensitive lipase is dephosphorylatedE. Free fatty acid concentration is higher than normal one

Find out the vitamin derivative that is synthesized from cholesterol in humans:

A. 1,25-dihydroxycholecalciferolB. EstradiolC. ProgesteroneD. Cholesterol esterE. Testosterone

The inhibition of one liver enzyme in cholesterol synthesis was after surplus intake of fat meat. Name this enzyme:

A. Beta-hydroxy-beta-methyl-glutaryl-CoA-reductaseB. Squalene oxygenaseC. Isopentenyl pyrophosphate isomeraseD. Mevalonate kinaseE. Acetyl - CoA- acetyl transferase

Choose the factor that can cause the 7-dehydrocholesterol conversion into vitamin D3 (in skin, only):

A. The accumulation of cholesterolB. UV-lightC. Low levels of oxygen in the bloodD. Cytochrome P450E. Hydroxylase action

Liebermann-Burchardt reaction is used for the identification of :

A. ProstaglandinsB. UreaC. CholesterolD. GlucoseE. Galactose

Bile acids are formed from:

A. Amino acidsB. BilirubinC. CholesterolD. ProteinE. Lipoproteins

Choose the key metabolite that combines carbohydrates catabolic pathways with anabolic processes in lipids metabolism:

A. SerineB. Acetyl~SCoAC. AlanineD. MevalonateE. Pyruvate

Point out the liver enzyme, which takes part in second step of transdeamination of amino acid:

A. Alpha-ketoglutarate dehydrogenaseB. Glutamate dehydrogenaseC. Tryptophan hydroxylaseD. L-Alanine oxidaseE. Glutamate decarboxylase

What characteristic of transamination is false?

A. It is transfer of an amino group from an amino acid to a keto acidB. It requires coenzyme derived from vitamin B<sub>2</sub>C. It is catalyzed by a group of enzymes called transaminasesD. There is no free NH<sub>3</sub> liberated, only the transfer of amino group occursE. It is catalyzed by pyridoxal phosphate dependent enzymes

Point out the cofactor, which is used by D-amino acid oxidase in oxidative deamination:

A. TPPB. FMNC. FADD. NADHE. NADPH

A patient diagnosed with carcinoma of bowels was admitted to the hospital. Analysis revealed high production of serotonin. It is known that this substance is formed of tryptophan amino acid. What biochemical mechanism underlies this process?

A. TransaminationB. Decarboxylation

C. DesaminationD. Formation of paired compoundsE. Microsomal oxidation

Glutamate dehydrogenase is controlled by allosteric regulation. Point out the inhibitors of this enzyme:

A. AMP, ADPB. ADP, GDPC. NAD<sup>+</sup>, NADP<sup>+</sup>D. ATP, GTPE. NADH, NADPH

Norepinephrine is synthesized (in mammals) from:

A. TyrosineB. ArginineC. TryptophanD. TryptamineE. Pyruvate

Point out the liver enzyme catalyzing the reversible oxidative deamination:

A. Alanine transaminaseB. ArginaseC. Aspartate transaminaseD. Monoamino oxidaseE. Glutamate dehydrogenase

The most important enzyme involved in oxidative deamination is:

A. HistidaseB. Glutamate dehydrogenaseC. L-amino acid oxidaseD. Amino acid dehydraseE. D-amino acid oxidase

Glutamate decarboxylation results in the formation of inhibitory transmitter in CNS. Name it:

A. HistamineB. SerotoninC. AsparagineD. GlutathioneE. Gamma amino butyric acid

Aspartate transaminase is the most abundant in:

A. BloodB. SpleenC. KidneyD. BrainE. Myocardium

Immediate products of pyruvate metabolism (using one reaction only) are all except:

A. Acetyl-CoAB. 2-PhosphoglycerateC. LactateD. OxaloacetateE. Alanine

Choose an amino acid that isn’t usually involved in direct deamination in humans:

A. Threonine

B. Aspartic acidC. SerineD. Glutamic acidE. Cysteine

Name the ketogenic amino acids:

A. Glutamate, aspartateB. Alanine, arginineC. Arginine, asparagineD. Cysteine, glycineE. Leucine, lysine

Choose an organic compound that can be converted due to oxidative deamination into &alpha;-ketoglutarate:

A. ThiamineB. GlucoseC. AlanineD. PyruvateE. Glutamic acid

The entry point into the citric acid cycle for isoleucine, valine, and the product of odd-chain fatty acids beta-oxidation is:

A. CitrateB. FumarateC. PyruvateD. Succinyl CoAE. Oxaloacetate

Which of the following descriptions related to serotonin pathway isn’t correct?

A. Serotonin synthesis lowering in the brain causes depressionB. The second enzyme of the pathway is pyridoxal phosphate-dependent aromatic amino acid decarboxylaseC. The largest amount of serotonin is synthesized in the intestinal cellsD. The first enzyme of the pathway is tetrahydrobiopterin-dependent hydroxylaseE. Serotonin is synthesized from tyrosine

Human Glutamate dehydrogenase is predominantly located in the:

A. Endoplasmic reticulumB. Inner mitochondrial membraneC. Mitochondrial matrixD. Outer mitochondrial membraneE. Cytosol

Choose the enzyme of the blood plasma whose activity increases in ten or more times for 3-4 hours after myocardium infarction:

A. Alanine transaminaseB. Alkaline phoshataseC. Aspartate transaminaseD. Leucine aminopeptidaseE. Arginase

The tested enzyme is related to the class of oxidoreductases and contains the coenzyme NADPH. It takes part in the oxidative deamination of one of the amino acids and in the reductive amination of alpha–ketoglutarate. Point out it:

A. Glutamate dehydrogenaseB. Arginase

C. D-Alanine oxidaseD. Alanine transaminaseE. Aspartate transaminase

Choose two amino acids that are not glucogenic:

A. Alanine, arginineB. Aspartate, glutamateC. Leucine, lysineD. Cysteine, methionineE. Serine, threonine

Most amino acids undergo transamination to concentrate finally nitrogen in two amino acids only for subsequent urea formation. Choose that amino acids:

A. Lysine, prolineB. Arginine, histidineC. Glutamate, aspartateD. Alanine, phenylalanineE. Serine, tyrosine

Choose the enzyme, whose genetic defect results in the GABA (Gamma-Amino Butyric Acid) levels decrease in the brain:

A. Glutamate decarboxylaseB. Phenylalanine hydroxylaseC. Histidine decarboxylaseD. Tryptophan decarboxylaseE. Alanine hydroxylase

Utilization of amino acids from the body pool is possible in all following ways except one. Choose it:

A. Pyridoxal phosphate synthesisB. Biogenic amine formationC. Porphyrins productionD. Purines productionE. Glucose and ketone bodies synthesis

Which of the following amino acids can undergo non-oxidative deamination?

A. GlutamateB. PhenylalanineC. HistidineD. AlanineE. Aspartate

Direct deamination is possible for two amino acids due to dehydratases (because of those amino acids side chain hydroxyl group is a good leaving group). Point out the amino acids:

A. Serine, threonineB. Cysteine, methionineC. Glutamate, aspartateD. Arginine, histidineE. Phenylalanine, tyrosine

Name biogenic amine that is formed from 5-hydroxy-tryptophan:

A. AdrenalinB. SerotoninC. ThiamineD. HistamineE. Dopamine

Which of the following statements related to biological role of transamination is wrong?

A. Collection finally nitrogen in glutamate for subsequent deamination and urea synthesisB. Formation of biogenic aminesC. Production of non-essential amino acidsD. Divergence the excess amino acids towards energy generationE. Redistribution of amino groups

Point out the way of of amino acids transformation that is not the common catabolic pathway:

A. HydroxylationB. TransaminationC. Alpha -decarboxylationD. TransdeaminationE. Oxidative deamination

Alpha-decarboxylation yields a vasodilator from:

A. ArginineB. SerineC. HistidineD. AspartateE. Glutamate

What characteristic of transamination is false?

A. It requires coenzyme derived from vitamin B6B. There is free NH<sub>3</sub> liberated, and &alpha;-keto acid is formedC. It’s transfer of an amino group from an amino acid to a keto acidD. It is catalyzed by pyridoxal phosphate dependent enzymesE. It is catalysed by a group of enzymes called transaminases

All of the following amino acids are converted to succinyl-CoA, except:

A. IsoleucineB. HistidineC. MethionineD. ValineE. All the proposed options are correct

The coenzyme that participates in transamination reaction is:

A. Thiamine pyrophosphateB. Coenzyme AC. Pyridoxal phosphateD. Coenzyme QE. NADPH

NADPH is used for next application except:

A. Glutamate synthesisB. Glucose synthesisC. Glutathione defense system against injury by reactive oxygen speciesD. Cholesterol synthesisE. Fatty acid biosynthesis

Point out the glucogenic amino acids:

A. GlutamateB. AlanineC. SerineD. AspartateE. All the positions are right

Utilization of amino acids from the body pool is possible in all following ways except one. Choose it:

A. Hormones and neurotransmitters formationB. Purines productionC. Creatine synthesisD. Porphyrins productionE. Pantothenic acid synthesis

The carbon skeletons of amino acids finally have one or more of the following fates except:

A. Oxidation via Krebs cycle to produce energyB. Synthesis of ureaC. Formation of fatty acid and ketone bodiesD. Synthesis of glucoseE. Synthesis of non-essential amino acids

Choose a substance that can be converted due to transamination into amino acid alanine:

A. Stearic acidB. Acetoacetic acidC. Linoleic acidD. Glutamic acidE. Pyruvic acid

Point out the enzyme, whose activity is determined in the blood plasma during the unicteric period of viral hepatitis:

A. Ornithine carbomoylphoshate transferaseB. Alanine transaminaseC. Creatine phosphokinaseD. Glutamate dehydrogenaseE. Phenylalanine hydroxylase

Hydrogen peroxide is a product of reaction catalysed by:

A. FumaraseB. CatalaseC. Lactate dehydrogenaseD. L-amino acid oxidaseE. Hexokinase

Which of the following statements related to biological role of transamination is wrong?

A. Production of non-essential amino acidsB. Redistribution of amino groupsC. Formation of biogenic aminesD. Divergence the excess amino acids towards energy generationE. Collection finally nitrogen in glutamate for subsequent deamination and urea synthesis

D-amino acid oxidase is important for the convertion of unnatural (for human body but regularly taken in the diet from plants) D—amino acids to L-amino acids. What prosthetic group is non-protein part of the enzyme?

A. FADB. NADHC. NADPHD. Pyridoxal phosphateE. TPP

What characteristic of transamination is false?

A. It’s transfer of an amino group from an amino acid to a keto acid

B. It is catalysed by a group of enzymes called transaminasesC. It requires coenzyme derived from vitamin B<sub>2</sub>D. It is catalyzed by pyridoxal phosphate dependent enzymesE. There is no free NH<sub>3</sub> liberated, only the transfer of amino group occurs

In course of histidine catabolism a biogenic amine is formed that has powerful vasodilatation effect. Name it:

A. HistamineB. SerotoninC. DopamineD. DioxyphenylalanineE. Noradrenalin

Point out the vitamin, whose deficiency causes the violations in the transamination and decarboxylation of amino acids:

A. Vitamin B<sub>9</sub>B. Vitamin B<sub>1</sub>C. Vitamin B<sub>2</sub>D. Vitamin B<sub>6</sub>E. Vitamin C

Most amino acids undergo transamination to concentrate finally nitrogen in two amino acids, only, for subsequent urea formation. Choose these amino acids:

A. Alanine, phenylalanineB. Glutamate, aspartateC. Serine, tyrosineD. Arginine, histidineE. Lysine, proline

Which of the following descriptions related to serotonin pathway isn’t correct?

A. Serotonin is synthesized from thyptophanB. Serotonin synthesis lowering in the brain causes depressionC. The second enzyme of the pathway is biotin-dependent aromatic amino acid decarboxylaseD. The first enzyme of the pathway is tetrahydrobiopterin-dependent hydroxylaseE. The largest amount of serotonin is synthesized in the intestinal cells

Histamine is produced from histidine by

A. Reduction and DecarboxylationB. Oxidation and DeaminationC. Decarboxylation onlyD. Reduction and AminationE. Deamination and Decarboxylation

According to clinical indications a patient was administered pyridoxal phosphate. What process is this medication intended to correct?

A. Deamination of purine nucleotideB. Transamination and decarboxylation of amino acidsC. Protein synthesisD. Synthesis of purine and pyrimidine bases.E. Oxidative decarboxylation of ketoacids

Which of the following coenzymes serves as a non-protein part of amino acid decarboxylase?

A. NADHB. NADPHC. Coenzyme AD. Pyridoxal phosphate

E. Biotin

Which of the following statements is incorrect concerning synthesis of carbamoyl phosphate?

A. Two ATP may be consumed for its synthesisB. It is the key reaction for urea formationC. The nitrogen source can be either ammonia or glutamineD. This synthesis is catalyzed by ornithine transcarbamoylaseE. It is mediated by different enzymes in the cytosol and mitochondria of the liver

The urine of patient with alkaptonuria contains:

A. PhenylalanineB. Homogentisic acidC. AcetatesD. None of the aboveE. Ketones

Point out the amino acid which serves as a carrier of ammonia from skeletal muscle to liver mainly:

A. SerineB. GlutamineC. ArginineD. AlanineE. Methionine

Alkaptonuria is due to the deficiency of one enzyme in human tissues. Choose it:

A. TyrosinaseB. DOPA oxidaseC. Phenylalanine hydroxylaseD. Homogentisate oxydaseE. Dihydroxyphenylalanine (DOPA) decarboxylase

The child has the genetic defect of one enzyme. It was proved by the appearance of phenylpyruvate in the urine. Point out this enzyme:

A. 5-tryptophan hydroxylaseB. Glycine amidaseC. Proline hydroxylaseD. Tyrosine hydroxylaseE. Phenylalanine 4-monooxygenase

The amino acids required for the formation of glutathione are:

A. Valine, leucine, isoleicineB. Glutamate, aspartate, arginineC. Glycine, cysteine, glutamateD. Cysteine, methionine, prolineE. Serine, Tyrosine, tryptophan

Urea is produced by single enzyme. Point out it:

A. GlutaminaseB. Carbamoyl phosphate synthetaseC. ArginaseD. UricaseE. Urease

The most direct precursor of taurine is:

A. TyrosineB. Glycine

C. CysteineD. MethionineE. Glutathione

Urea synthesis requires five enzymes. Which enzyme is not related to urea cycle?

A. GlutaminaseB. Argino-succinate synthaseC. Argino-succinaseD. Carbamoyl phosphate synthase IE. Ornithine transcarbamoylase

Tetrahydrobiopterin is involved in the synthesis of all compounds placed below except:

A. EpinephrineB. MelatoninC. PhenylalanineD. TyrosineE. 5-hydroxytryptophan

The metabolic fate of phenylalanine may be various in human body. Point out a product that couldn't be formed from phenylalanine:

A. TyrosineB. ThyroxineC. EpinephrineD. MelaninE. Serotonin

Point out the cofactor required for the tyrosine formation from phenylalanine:

A. Lipoic acidB. Coenzyme QC. Coenzyme AD. TetrahydrobiopterinE. Tetrahydrofolate

Point out the vitamin derivative that may be synthesized from tryptophan:

A. CarboxybiotinB. NADHC. FADD. CoASHE. FMN

Which of the following compounds serves as a primary link between the citric acid cycle and the urea cycle?

A. SuccinateB. MalateC. CitrateD. IsocitrateE. Fumarate

The deficiency of this coenzyme causes certain symptoms consistent with a diagnosis of phenylketonuria. Point out the coenzyme:

A. TetrahydrobiopterinB. LipoamideC. Pyridoxal phosphateD. Thiamine pyrophosphateE. NADPH

Albinism is due to deficiency of the following enzyme:

A. Homohentisic acid oxidaseB. TyrosinaseC. Phenylalanine hydroxylaseD. Tyrosine decarboxylaseE. Tyrosine transaminase

Choose the enzyme used for tyrosine formation from phenylalanine:

A. Phenylalanine-4-hydroxylaseB. Aspartate transaminaseC. TyrosinaseD. Dioxyphenylalanine decarboxylaseE. Phenylalanine transaminase

Maple syrup urine disease is due to deficiency of:

A. DeaminaseB. DecarboxylaseC. DehydroxylaseD. Alpha-keto acid dehydrogenase complexE. Transaminase

After a serious viral infection a 3-year-old child has repeated vomiting, loss of consciousness, convulsions. Examination revealed hyperammoniemia. What may have caused changes of biochemical blood indexes of this child?

A. Disorder of ammonia neutralization in ornithine cycleB. The inhibited activity of enzymes for transaminationC. Activated processes of amino acids decarboxylationD. Disorder of biogenic amines neutralizationE. The increased putrefaction of proteins in intestines

Which of the following is NOT an enzyme of Urea synthesis:

A. Carbamoyl phosphate synthase IIB. Ornithine synthaseC. Argininosuccinate lyaseD. Carbamoyl phosphate synthase IE. Arginase

The following amino acid is essential for synthesis of nitric oxide:

A. ArginineB. GlycineC. AlanineD. AspartateE. Glutamate

The brain ammonia isn’t neutralized through the formation of urea. Name the terminal product of ammonia neutralization in the brain:

A. Carbamoyl phosphateB. Aspartic acidC. AlanineD. UreaE. Glutamine

Which of the following compounds is not a catabolic product of tryptophan?

A. TryptamineB. Melatonin

C. KynurenineD. MelaninE. Serotonin

Citrullinemia accompanied with vomiting, mental retardation, convulsions. Which urea cycle enzyme hereditary defect is a cause of mentioned disturbances?

A. Arginosuccinate synthaseB. ArginaseC. Ornithine transcarbamoylaseD. Carbamoyl phosphate synthase IE. Arginosuccinase

Point out the pathology that is estimated in patients with genetic deficiency of dihydrobiopterin reductase:

A. Ishemic heart diseaseB. Classic phenylketonuriaC. GoutD. Phenylketonuria type IIE. Diabetes mellitus

N-acetylglutamate is activator for one enzyme involved in urea formation. Choose it:

A. Argino-succinaseB. Carbamoyl phosphate synthase IC. Argino-succinate synthaseD. Ornithine transcarbamoylaseE. Arginase

A 4 y.o. child with signs of durative proteinic starvation was admitted to the hospital. The signs were as follows: Growth inhibition, anemia, oedema, mental deficiency. Choose the cause of oedema development:

A. Reduced synthesis of lipoproteinsB. Reduced synthesis of albuminsC. Reduced synthesis of glycoproteinsD. Reduced synthesis of globulinsE. Reduced synthesis of hemoglobin

Which of the following is correct concerning synthesis of carbamoyl phosphate?

A. The nitrogen source can be either ammonia or glutamine.B. All positions are correct.C. It takes place in pyrimidine synthesis.D. It is mediated by different enzymes in the cytosol and mitochondria of the liver.E. It is the key reaction for urea formation

Point out the coenzyme used for hydroxylases structure in phenylalanine and tyrosine conversions:

A. TetrahydrobiopterinB. BiotinC. NADPHD. FADE. Dihydrobiopterin

Choose the hormone that is formed from tryptophan:

A. Nor-epinephrineB. HistamineC. ThyroxinD. EpinephrineE. Melatonin

Point out the enzyme in urea cycle used for arginine formation:

A. Aspartate transaminaseB. Arginine transaminaseC. Ornithine transcarbamoylaseD. Argininosuccinate lyaseE. Arginase

Which of the following statements is incorrect concerning synthesis of carbamoyl phosphate?

A. Mitochondrial carbamoyl phosphate synthase requires aspartate for its activityB. The nitrogen source can be either ammonia or glutamineC. It is the key reaction for urea formationD. This synthesis is catalyzed by carbamoyl phosphate synthasesE. Two ATP may be consumed for its synthesis

Name an amino acid which is precursor in the synthesis of some hormones in human body:

A. TyrosineB. ArginineC. LysineD. AlanineE. Serine

Which metabolite of Citric Acid Cycle is used in detoxification of ammonia in the brain:

A. OxaloacetateB. FumarateC. Alpha ketoglutarateD. MalateE. Aconitate

Low level of albumins and fibrinogen was detected in the patient's blood. Decreased activity of what organelle in the hepatocyte can most probably cause it?

A. Granular endoplasmatic reticulumB. LysosomesC. Golgi complexD. Agranular endoplasmatic reticulumE. Mitochondrions

Ammonia is detoxified in the liver to form mainly:

A. Uric acidB. AlanineC. UreaD. GlutamineE. Creatinine

Albinos can't stand sun impact – they don't require sun-tan but get sunburns. Disturbed metabolism of what amino acid underlies this phenomenon?

A. TryptophanB. MethionineC. PhenylalanineD. HistidineE. Glutamic acid

Melanin may be synthesized from:

A. PhenylalanineB. TryptophanC. Methionine

D. SerineE. Cysteine

Choose the amino acid which may be converted into a vitamin derivative:

A. GlycineB. LysineC. PhenylalanineD. TryptophanE. Leucine

Which of the following is not synthesized from tyrosine:

A. DopamineB. MelatoninC. ThyroxineD. EpinephrineE. Norepinephrine

Phenylketonuria is due to the absence of certain enzyme. Choose the reaction catalyzed by the enzyme:

A. Tyrosine iodinationB. Phenylalanine hydroxylationC. Homogentisic acid oxidationD. Tyrosine deaminationE. Conversion of tyrosine to dihydroxyphenylalanine (DOPA)

Which one of the following amino acids can be converted to an intermediate of either the Krebs cycle or urea cycle?

A. AspartateB. TryptophanC. LysineD. TyrosineE. Leucine

Choose the type of infringement in pathology named alkaptonuria:

A. The renal insufficiencyB. The inhibition of amino acid formationC. The viral damage of hepatocyteD. The damage of receptor synthesisE. Genetic deficiency of enzyme

Alkaptonuria is due to the deficiency of one enzyme in human tissues. Choose it

A. Dihydroxyphenylalanine (DOPA) decarboxylaseB. DOPA oxidaseC. Phenylalanine hydroxylaseD. Homogentisate oxydaseE. Tyrosinase

Point out an amino acid not involved in urea synthesis.

A. OrnithineB. CitrullineC. Aspartic acidD. HistidineE. Arginine

Maple syrup urine disease is associated with a defect in oxidative decarboxylation of the following amino acid:

A. Methionine and serineB. Proline and hydroxyprolineC. Leucine and isoleucineD. PhenylalanineE. Cysteine and cystine

A newborn child was found to have reduced intensity of sucking, frequent vomiting, hypotonia. Urine and blood exhibit increased concentration of citrulline. What metabolic process is disturbed?

A. GlycolysisB. Cori cycleC. GlyconeogenesisD. Tricarboxylic acid cycleE. Ornithinic (Urea) cycle

Ammonia is a very toxic substance, especially for the nervous system. What substance takes the most active part in ammonia detoxification in the brain tissue?

A. AlanineB. Glutamic acidC. HistidineD. LysineE. Proline

Cerebral trauma caused the increase of ammonia formation. What amino acid takes part in removal of ammonia from cerebral tissue?

A. TryptophanB. ТуrosineC. LysineD. ValineE. Glutamic acid

Find out the main important pathway for ammonia utilization in the brain:

A. Amino acid decarboxylationB. Urea formationC. Ammonia salts formationD. Conversion into glucoseE. Synthesis of glutamine from alpha-ketoglutarate

Which of the following functions is not specific for glutathione:

A. It prevents the oxidation of sulfhydryl groups of several proteins to disulfide groupsB. It is involved in the transport of amino acids across cellular membrane in the intestine and kidneyC. It protects a cell against cytotoxic H<sub>2</sub>O<sub>2</sub> being coenzyme of glutathione peroxidaseD. It serves as coenzyme for certain enzymes e.g. prostaglandine PGE<sub>2</sub> synthaseE. It serves as a storage and transport form of NH<sub>3</sub>

A boy (of 10 years) complains of general weakness, dizziness, and tiredness. A mental retardation is observed. A concentration of valine, leucine, isoleucine is high in blood and urine. Urine has a specific odour. Name the probable diagnosis:

A. HistidinemiaB. Basedow’s diseaseC. TyrosinemiaD. Maple syrup urine diseaseE. Addison’s disease

Choose the amino acid that may be used for utilization of ammonia as acceptor of ammonia molecule:

A. Glutamic acid

B. CysteineC. AlanineD. ValineE. Threonine

Maple syrup urine disease is due to a defect in the enzyme complex which catalyzes oxidative decarboxylation reaction and is similar to any other &alpha;-keto acid dehydrogenase complexes in composition. Some patients who exhibit the symptoms of this disease respond to therapeutic doses of hereinafter stated vitamins. Point out it:

A. ThiamineB. BiotinC. CobalamineD. Ascorbic acidE. Pyridoxine

Xanthurenic acid is a product of metabolism of:

A. XanthineB. Glucuronic asidC. TryptophanD. UreaE. Uric acid

One of hereinafter stated compounds forms a connecting link between urea cycle and Krebs cycle via oxaloacetate. Point out it:

A. AlanineB. TryptophanC. AspartateD. PhenylalanineE. Glutamate

If urine sample darkens on standing, most likely diagnosis for patient is:

A. AlkaptonuriaB. AlbinismC. Maple syrup urine deseaseD. Diabetes mellitusE. Phenylketonuria

Try to choose the index of blood plasma used for estimation of liver function in utilization of ammonia:

A. Urea contentB. Glucose concentrationC. Pyruvic acid contentD. Ketone bodies contentE. Uric acid concentration

A 4 y.o. boy has had recently serious viral hepatitis. Now there are such clinical symptoms as vomiting, unconsciousness, fits. There is hyperammoniemia in patient, too. Disturbance of which biochemical process caused such pathological condition of the patient?

A. Disturbed neutralization of ammonia in the liverB. Activation of amino acid decaboxylationC. Disturbed neutralization of biogenic aminesD. Inhibition of transamination enzymesE. Increased putrefaction of proteins in bowels

Albinism is due to the lack of special enzyme in melanocytes. Name it:

A. Tyrosine decarboxylaseB. Tyrosinase

C. Phenylalanine hydroxylaseD. KynureninaseE. Homogentisate oxydase

Labeled by radioisotope amino acids alanine and tryptophan were administrated to a mouse in order to study the localization of protein synthesis in cells. Around what organelles in the cell the accumulation of labeled amino acids will be observed?

A. RibosomesB. Golgi apparatusC. Agranular endoplasmic reticulumD. LysosomesE. Cell nucleus

Choose the blood plasma index that is used in screening of newborn for phenylketonuria estimation:

A. Uric acidB. PyruvateC. DihydroxyphenylalanineD. PhenylalanineE. Acetoacetate

The major source of ammonia in the kidneys is:

A. GlutamateB. UreaC. AspartateD. GlutamineE. Histidine

Glycine is used for synthesis of:

A. PurineB. HemoglobinC. GlutathioneD. All the proposed options are correctE. Creatine

Nappies of a newborn have dark spots that witness the presence of homogentisic acid oxidation product. Choose the substance whose metabolic disorder is associated with accumulation of homogentisic acid in the organism:

A. MethionineB. GalactoseC. TyrosineD. TryptophanE. Cholesterol

One amino acid that is not involved in the urea cycle is:

A. CitrullineB. OrnithineC. ArginineD. HistidineE. Aspartate

Point out the enzyme whose deficiency causes the hyperammonemia state in patient:

A. L-Alanine oxidaseB. Aspartate transaminaseC. Ornithine transcarbamoylaseD. Arginine transaminaseE. Pyruvate dehydrogenase

Three biogenic amines are formed during the metabolism of tyrosine. Point out one of them:

A. ThiamineB. HistamineC. AdrenalinD. ThyroxinE. Serotonin

Point out the index of the blood plasma that is used for the estimation of liver parenchyma damage:

A. Ca<sup>2+</sup>B. Cholesterol esterC. GlucoseD. Free amino acidsE. Urea

Point out the pathologic state that may be estimated in patients with genetic defect of Phenylalanine -4-hydroxylase:

A. GlucosemiaB. DislipoproteinemiaC. AminoaciduriaD. HyperuricemiaE. Hypercholesterinemia

The substance from which ammonia is produced by the kidney is:

A. GlutamineB. AlanineC. MethionineD. CytosineE. Glycine

Choose the enzyme that takes part in the gastric digestion of carbohydrates:

A. Amylo-1,6-glycosidaseB. Salivary amylaseC. SucraseD. There's no any enzyme for carbohydrates digestion in stomachE. Oligo-1,6-glycosidase

Name the enzyme whose function is associated with digestion of polysaccharides in the small intestine:

A. MaltaseB. PepsinogenC. ElastaseD. Alpha-amylaseE. Rennin

Point out the class of enzymes that catalyze the digestion of proteins in gastro-intestinal tract:

A. LigasesB. TransferasesC. OxidoreductasesD. HydrolasesE. Lyases

Point out the couple of amino acids participating in the formation of peptide bond that is cleaved by trypsin:

A. Leucine, valineB. Glycine, Glutamine

C. Isoleucine, alanineD. Alanine, valineE. Arginine, lysine

The vegetable oils are the obligatory components of man ration as they contain some essential fatty acids. Choose them:

A. Acetic and butyric acidsB. Stearic and palmitic acidsC. Citric and fumaric acidsD. Linoleic and linolenic acidsE. Palmitooleic and oleic acids

After intake of rich food a patient feels nausea and sluggishness; with time signs of steatorrhea there appeared. Blood cholesterol concentration is 9.2 mmole/l. That condition was caused by lack of:

A. ChylomicronsB. PhospholipidsC. Bile acidsD. Fatty acidsE. Triglycerides

A 30-year-old woman was diagnosed with insufficiency of exocrine function of pancreas. Hydrolysis of what nutrients will be disturbed?

A. Proteins, fats, carbohydratesB. Proteins, carbohydrates, onlyC. Proteins, fats, onlyD. Proteins, onlyE. Fats, carbohydrates, only

The bile acids participate in:

A. The activation of trypsinB. The protein transport activationC. The lipids emulsificationD. The cholesterol synthesisE. The ketone bodies synthesis

Point out the endopeptidase that is produced by pancreas and is activated by trypsin:

A. ProelastaseB. Alpha-AmylaseC. GastricsinD. PepsinogenE. Renin

Point out the activator of secretin production in duodenum:

A. ChymotrypsinB. GastricsinC. TrypsinD. PepsinE. Hydrochloric acid

Trypsin is found in functions as activator for some zymogens produced by pancreas. Choose them:

A. Procarboxylase AB. All that is placedC. Procarboxylase BD. ProelastaseE. Chymotrypsinogen

The stone of a general bile duct stopped the entrance of bile to the intestine of the patient. Specify the process, whose infringement will be observed:

A. Protein digestionB. Fats digestionC. Carbohydrates digestionD. Carbohydrates absorptionE. Protein digestion

Point out the group of peptidases which trypsin is belong to:

A. ExopeptidaseB. EndopeptidaseC. DipeptidaseD. Amino peptidaseE. Carboxypeptidase

A patient complains of frequent diarrheas, especially after consumption of fattening food and of the body weight loss. Laboratory examination revealed steatorrhea; hypocholic feces. What may be the cause of this condition?

A. Obturation of biliary tractsB. Unbalanced dietC. Lack of pancreatic lipaseD. Lack of pancreatic phospholipaseE. Mucous membrane inflammation of small intestine

This disorder may be genetic or acquired. Affected infants are not able to tolerate milk, which is normally their primary food. Point out the name of disorder:

A. Lactose intoleranceB. Glucose intoleranceC. Galactose intoleranceD. FructosuriaE. Galactosemia

Find out the values for total acidity of gastric juice associated with hypochlorhydria in patient:

A. 40 mmol/LB. 20 mmol/LC. 50 mmol/LD. 60 mmol/LE. 55 mmol/L

Point out the amino acids whose peptide bonds are hydrolyzed by pepsin and chymotrypsin:

A. Diamino acidsB. Aromatic amino acidsC. Alcoholic acidsD. Amides of monocarboxylic acidsE. Sulfur-containing amino acids

Examination of a man who hadn’t been consuming fats but had been getting enough carbohydrates and proteins for long time revealed dermatitis, poor wound healing, vision impairment. What is the probable cause of metabolic disorder?

A. Low caloric value of dietB. Lack of vitamins PP, HC. Lack of palmitic acidD. Lack of linoleic acid, vitamins A, D, E, K.E. Lack of oleic acid

Lipase requires special conditions for its activity. Choose a factor that isn’t optimum for this purpose:

A. ColipaseB. PhospholipidsC. pH=8.0D. Bile saltsE. pH=2.0

Achlorhydria (achylia) state is observed in patient with stomach problems. Propose probable reason for the development of this state and probable values for free HCL content in the gastric juice of this patient:

A. Colitis; [free HCL]=60 mmole/LB. Obstruction of esophagus; [free HCL]=20 mmole/LC. Extensive secretion of gastric juice; [free HCL]=60 mmole/LD. Disturbed pepsinogen synthesis; [free HCL]=10 mmole/LE. Cancer of stomach; [free HCL]= 0 mmole/L

Some carbohydrates can't be digested in the intestine. Point out one of them:

A. SucroseB. MaltoseC. GlycogenD. StarchE. Cellulose

The only sugar absorbed by intestine against a concentration gradient is:

A. GlucoseB. MannoseC. GalactoseD. FructoseE. Xylose

There is one organic compound in the list below to inhibit phosphofructokinase I if this compound is accumulated in cytoplasm. Name it:

A. PyruvateB. SuccinateC. Fructose-6-phosphateD. Glyceroaldehyde 3-phosphateE. ATP

Which one of the following enzymes catalyzes phosphorylation with the use of inorganic phosphate?

A. HexokinaseB. Glyceraldehyde-3-phosphate dehydrogenaseC. Pyruvate kinaseD. PhosphofructokinaseE. Phosphoglycerate kinase

The functions of gluconeogenesis are described by all of the following statements EXCEPT:

A. It allows the use of amino acids for glucose productionB. It maintains blood sugar levels during fastingC. It is useful during strenuous exercisesD. It allows the use of acetyl-CoA for glucose productionE. It maintains blood glucose level during period of limited carbohydrate intake

A deficiency in the vitamin Biotin leads to a deficiency in the activity of certain enzyme of gluconeogenesis. Point out the enzyme:

A. Phosphoglycerate kinaseB. PhosphoenolpyruvatecarboxykinaseC. Pyruvate carboxylase

D. Glucose 6-phosphataseE. Fructose-1,6-bisphosphatase

Which statement about gluconeogenesis is correct?

A. It occurs primarily in skeletal muscle.B. Lactate and alanine can both serve as substrates.C. Phosphofructokinase I is the rate-limiting enzyme of gluconeogenesis.D. The acetate group of acetyl-CoA is used for the net synthesis of glucoseE. ATP is not required for the process

The intensive muscular work increases the velocity of the ATP utilization and causes the activation of glycolysis. What enzyme activity will limit its rate?

A. PhosphoglyceromutaseB. AldolaseC. PhosphofructokinaseD. Glucose-6-phosphate isomeraseE. Phosphoglycerate kinase

Name the factors which are important to regulate the anaerobic glycolysis duration:

A. Fructose-2.6-biphosphate levelB. NADH/NAD[sup]+[/sup] ratio in a cellC. All the factors mentionedD. ATP/ADP ratio in a cellE. Oxygen level in tissue

Choose the key (regulatory) enzymes of glycolysis:

A. Fructose-1,6-bisphosphatase, phosphoenolpyruvatecarboxykinase, pyruvate carboxylaseB. Glucokinase, phosphotrioseisomerase, lactate dehydrogenaseC. Phosphohexoseisomerase, phosphoglycerate kinase, aldolase ID. Citrate synthase, isocitratedehydrohenase, alpha-ketoglutarate dehydrogenaseE. Hexokinase, phosphofructokinase, pyruvate kinase

Choose the energy requirement (sum total of ATP; to think about GTP use as ATP use) for glucose synthesis from 2 moles of pyruvic acid:

A. 6 ATPB. 4 ATPC. 1 ATPD. 2 ATPE. 3 ATP

The activity of pyruvate carboxylase is dependent upon the positive allosteric effector:

A. AMPB. Acetyl CoAC. CitrateD. SuccinateE. Isocitrate

Choose the yields a net for 1 mole of glucose oxidation by anaerobic glycolysis:

A. 2 moles of pyruvate, 2 moles of NADH, 2 moles of ATPB. 2 moles of lactate and 6 moles of ATPC. 2 moles of pyruvate and 2 moles of ATPD. 2 moles of lactate and 2 moles of ATPE. 2 moles of lactate, 2 moles of NAD+, 2 moles of ATP

What enzyme catalyzes the glucose-6-phosphate formation from glucose in the liver and is not inhibited by excess level of glucose-6-phosphate:

A. Glucose-6-phosphataseB. GlucokinaseC. Pyruvate kinaseD. PhosphoglucomutaseE. Hexokinase

Choose the enzyme for the reaction of glucose formation due to dephosphorylation:

A. GlucokinaseB. Glucose-6-phosphataseC. AldolaseD. HexokinaseE. Phosphofructokinase

Gluconeogenesis occurs in the liver and:

A. SpleenB. KidneyC. Adipose tissueD. Skeletal muscleE. Heart

Choose the condition in human organism which can cause the beginning of gluconeogenesis in the liver:

A. HyperglycemiaB. The decrease of diuresisC. HypoglycemiaD. The bile ducts obstructionE. The hypoxia of liver tissue

Name, please, the liver enzyme needed for lactate utilization in gluconeogenesis:

A. NADH-dependent Lactate DehydrogenaseB. LactonaseC. Pyruvate dehydrogenaseD. E. Glucose-6-phosphataseF. NAD-dependent Lactate Dehydrogenase

Which compound does not contribute to the net formation of glucose during gluconeogenesis in man?

A. GlycerolB. LactateC. Glyceraldehyde 3-phosphateD. OxaloacetateE. Acetyl-CoA

Diseases of the respiratory system and circulatory disorders impair the transport of oxygen, thus leading to hypoxia. Under these conditions the energy metabolism is carried out by anaerobic glycolysis. As a result, the following substance is generated and accumulated in blood:

A. Citric acidB. Glutamic acidC. Pyruvic acidD. Fumaric acidE. Lactic acid

Which of the following compounds is an allosteric activator that enhances activity of phosphofructokinase I of the glycolytic pathway?

A. GlucoseB. Citric acid

C. Adenosine triphosphate (ATP)D. Adenosine monophosphate (AMP)E. Glucose-6-phosphate

The genetic defect of pyruvate carboxylase deficiency is the cause of delayed physical and mental development and early death in children. This defect is characterized by lacticemia, lactaciduria, disorders of a number of metabolic pathways. In particular, the following processes are inhibited:

A. Pentose phosphate pathway and glycolysisB. Citric acid cycle and gluconeogenesisC. Lipolysis and lipogenesisD. Glycolysis and glycogenolysisE. Glycogenesis and glycogenolysis

Find out the location of glucose-6-phosphatase in human tissues:

A. MyocardiumB. Liver, kidneyC. Gonads, onlyD. Skeletal muscular tissueE. Liver, only

Gluconeogenesis from lactate does not require activity of:

A. Glyceraldehyde 3-phosphate dehydrogenase.B. Triose phosphate isomerase.C. Aldolase.D. Phosphoglycerate kinase.E. Phosphofructokinase.

A 7-year-old girl has signs of anemia. Laboratory examination revealed pyruvate kinase deficiency in the erythrocytes. What process disturbance plays the main role in anemia development?

A. Tissue respirationB. Anaerobic glycolysisC. Peroxide decompositionD. Oxidative phosphorylationE. Amino acids desamination

The gluconeogenesis is activated in the liver after intensive physical trainings .What substance is utilized in gluconeogenesis first of all in this case:

A. GlucoseB. LactateC. PyruvateD. GlutamateE. Alanine

Choose the key enzymes of gluconeogenesis:

A. Phosphohexoseisomerase, phosphoglycerate kinase, aldolase IB. Pyruvate carboxylase, phosphoenolpyruvatecarboxykinase, fructose-1,6-bisphosphataseC. Glucokinase, phosphotrioseisomerase, lactate dehydrogenaseD. Hexokinase, phosphofructokinase, pyruvate kinaseE. Citrate synthase, isocitratedehydrohenase, alpha-ketoglutarate dehydrogenase

Name the energy effect of anaerobic glycolysis per 1 mole of glucose incorporated into the process:

A. 2 ATPB. 3 ATPC. 5 ATPD. 8 ATPE. 10 ATP

Which of the ions is the most important for glycolysis?:

A. CobaltB. ZinkC. MagnesiumD. CalciumE. Aurum

Which one of the following enzymes is common to both glycolysis and gluconeogenesis?

A. Phosphoglycerate kinaseB. HexokinaseC. Pyruvate kinaseD. Fructose-1,6-bisphosphataseE. Pyruvate carboxylase

A child has got galactosemia. Concentration of glucose in blood has not considerably changed. Deficiency of what enzyme caused this illness?

A. HexokinaseB. Galactose-1-phosphate uridyltransferaseC. PhosphoglucomutaseD. Amylo-l,6-glucosidaseE. Fructokinase

Which of the following compounds is required by transketolase for maximal activity?

A. Coenzyme AB. BiotinC. Thiamine pyrophosphateD. Acetyl-CoAE. Dihydroxyacetone phosphate

Pyruvate concentration in the patient’s urine is increased 10 times than the normal level. Choose the Vitamin, the deficiency of which in the organism can be the reason of this change:

A. Vitamin CB. Vitamin B[sub]1[/sub]C. Vitamin B[sub]6[/sub]D. Vitamin EE. Vitamin A

Point out the multienzyme system where enzymes form a single structural-functional complex:

A. Glucose-6-Phosphate dehydrogenaseB. C. Enzymes of Krebs cycleD. Glutamate dehydrogenaseE. Carbamoyl phosphate synthetaseF. Pyruvate dehydrogenase complex

Malate-aspartate shuttle system is required for:

A. Alanine transport into mitochondriaB. GluconeogenesisC. Pentose phosphate pathwayD. Embden-Meyerhoff pathwayE. Reducing equivalent (NADH) transport into mitochondria

NADPH is used for next application EXCEPT:

A. Cholesterol synthesis

B. Glutathione defense system against injury by reactive oxygen speciesC. Monooxygenase system linked with cytochrome P450D. Glucose synthesisE. Fatty acid biosynthesis

Which enzyme is present in the liver but not in muscle?

A. Glucose 6-phosphataseB. Pyruvate dehydrogenaseC. Glycogen phosphorylaseD. Lactate dehydrogenaseE. Hexokinase

Immediate products of pyruvate metabolism (using one reaction only) are all EXCEPT:

A. 2-PhosphoglycerateB. Acetyl-CoAC. LactateD. OxaloacetateE. Alanine

Point out the substrate that is used as major energy source for brain:

A. Fatty acidB. Ketone bodyC. GlucoseD. AlanineE. Phospholipid

Biochemical test in thiamine deficiency may be:

A. Ferric chloride testB. Cholesterol testC. Histidine testD. Glucose testE. Erythrocyte transketolase test

A child’s blood has high content of galactose, glucose concentration is low. There are such presentations as cataract, mental deficiency, adipose degeneration of liver. What disease is it?

A. LactosemiaB. GalactosemiaC. Steroid diabetesD. FructosemiaE. Diabetes mellitus

The aerobic oxidation of carbohydrates includes the oxidative decarboxylation of pyruvate. Specify the inhibitor (in high concentration) for this reaction:

A. ADPB. FADC. NADP[sup]+[/sup]D. Acetyl-SCoAE. NAD[sup]+[/sup]

Name the second stage of aerobic glucose oxidation in tissues:

A. Krebs CycleB. Pyruvate formationC. Acetyl-SCоA carboxylationD. Oxidative decarboxylation of pyruvateE. Alpha-ketoglutarate formation

Choose the INCORRECT statement concerning functions of the pentose phosphate pathway:

A. It is a source of NADPH for reductive pathwaysB. It is a route for the use of pentoses and for their conversion to hexoses and triosesC. It is a route for conversion of pentoses from degradated nucleotides to intermediates of glycolysisD. It is a source of pentoses for nucleic acids synthesisE. It is a source of ATP for reductive pathways

Sites where Hexose monophosphate shunt can occur include:

A. All the proposedB. White Blood Cells (WBC)C. TestesD. LiverE. Lactating mammary gland

Point out the multienzyme system used in aerobic oxidation of glucose up to carbon dioxide and water formation:

A. Glucose-6-Phosphate dehydrogenase complexB. Succinate dehydrogenase complexC. Alpha-ketoglutarate dehydrogenase complexD. Palmitate synthetaseE. Glutamate dehydrogenase complex

A child with galactosemia is referred to an opthalmologist for evaluation of developing cataracts, probably caused by the accumulation of galactose and its alcohol galactitol. Choose the enzyme catalyzing the transformation of galactose to galacitol:

A. GalactokinaseB. Aldolase BC. Galactose 1-phosphate uridyltransferaseD. GlucokinaseE. Aldose reductase

Pyruvate dehydrogenase complex contains all cofactors except one. Choose it:

A. BiotinB. FADC. CoASHD. NAD[sup]+[/sup]E. Lipoamide

How does epinephrine influence the glucose level in the blood?

A. Decreases, inhibiting the glycogen synthesis in the liverB. Decreases, stimulating gluconeogenesis in the liverC. Does not influenceD. Increases, stimulating glycogen breakdown in the liverE. Decreases, inhibiting the glycolysis in muscles

Point out the key enzyme of glycogen degradation in the liver:

A. Glycogen PhosphorylaseB. Glucose oxidaseC. Glyceraldehyde-3-phosphataseD. Glucose-6-phosphataseE. Fructose-1,6-bisphosphatase

McArdle’s disease development causes muscle cramps and muscle fatigue with increased muscle glycogen in a patient. Which of the following enzymes is deficient?

A. Hepatic hexokinase

B. Muscle debranching enzymeC. Muscle hexokinaseD. Muscle glycogen synthetaseE. Muscle phosphorylase

Inherited disease, such as mucopolysaccharidosis, is manifested in metabolic disorders of connective tissue, bone and joint pathologies. The sign of this disease is the excessive urinary excretion of the following substance:

A. LipidsB. Amino acidsC. GlycosaminoglycansD. UreaE. Glucose

How glucocorticoids influence the carbohydrate metabolism?

A. Stimulate the glycogenesisB. Stimulate the starch hydrolysis in the small intestineC. Stimulate the gluconeogenesisD. Inhibit the glycogen phosphorolysisE. Stimulate the glycolysis from glucose

WHICH one of the following enzymes is associated with glycogen synthesis?

A. Glucose-6-phosphataseB. Phosphorylase kinaseC. Debranching enzymeD. Glycogen phosphorylaseE. Branching enzyme

Which of the following metabolites is found in glycogenolysis, glycolysis, and gluconeogenesis?

A. Fructose-6-phosphateB. Uridine diphosphogalactoseC. Galactose-1-phosphateD. Uridine diphosphoglucoseE. Glucose-6-phosphate

After meals blood glucose enters liver cells and is stored as glycogen, mainly. Which of the following compounds is the donor of new glucose residue in glycogen?

A. UDP-glucose-1-phosphateB. Glucose-4-phosphateC. UDP-glucose-6-phosphateD. Glucose-6-phosphateE. UDP-glucose

Mucopolysaccharidoses are inherited storage diseases. They are caused by:

A. The synthesis of abnormally small amounts of protein coresB. Defects in the degradation of proteoglycansC. All the proposed options are correctD. An increased rate of synthesis of proteoglycansE. The synthesis of polysaccharides with an altered structure

Insulin stimulates all the processes listed below in adipocytes except:

A. GlycogenolysisB. Transport of glucose through cell membraneC. Oxidative decarboxylation of pyruvateD. Catabolism of glucoseE. Hexose monophosphate shunt

How does adrenalin influence the glucose level in the blood?

A. Decreases, inhibiting the glycolysisB. Increases, stimulating the glycogen destructionC. Does not influenceD. Decreases, stimulating the gluconeogenesisE. Decreases, inhibiting the glycogen synthesis

Which glycosaminoglycan is the most abundant in cartilage?

A. Hyaluronic acidB. Dermatan sulfateC. Keratan sulfateD. Heparan sulfateE. Chondroitin sulfate

Which of the following alterations would be expected to occur after a substantial breakfast?

A. Decreased rate of protein synthesisB. Decreased activity of glucokinaseC. Increased activity of pyruvate carboxylaseD. Decreased rate of glycogenolysisE. Increased activity of phosphoenolpyruvate carboxykinase

A child is languid, apathetic. Liver is enlarged and liver biopsy revealed a significant amount of glycogen. Glucose concentration in the blood stream is below normal. What is the cause of low glucose levels:

A. Low {absent} activity of glucose 6-phosphataseB. High activity of glycogen synthetaseC. Low {absent} activity of hexokinaseD. High activity of glycogen phosphorylase in liverE. Deficit of gene that is responsible for the synthesis of glucose 1-phosphate uridine transferase

What glycolytic enzyme synthesis is stimulated due to insulin:

A. AldolaseB. Hexokinase IVC. Phosphoglycerate kinaseD. Glucose-6-phosphataseE. Adenylate kinase

Choose the hormone whose secretion may be damaged at diabetes mellitus in person:

A. Growth hormoneB. ProgesteroneC. InsulinD. EpinephrineE. Cortisol

A child has been diagnized for low blood glucose (hypoglycemia), enlarged liver (hepatomegaly), and excess fat deposition in the cheeks. A liver biopsy reveals excess glycogen in hepatocytes. There is hypothetically Pompe’s desease. Deficiency of which of the following enzymes might explain this phenotype?

A. Glucosyl -4:6 transferase (branching enzyme)B. Lysosomal alpha-1,4-glucosidaseC. Alpha-1,4-galactosidaseD. Glucose 6-phosphataseE. Amylo alpha-1,6 glucosidase (debranching enzyme)

The tissue with the highest glycogen content (mg/100g of tissue) is:

A. KidneyB. TestesC. BrainD. Skeletal muscleE. Liver

The most important substances as a source of blood glucose after 48 hours of starvation are:

A. Fructose and galactoseB. Amino acidsC. Muscle and liver glycogenD. FructoseE. Liver glucoseaminoglycans

The investigation of the blood and urine of patient with diabetes mellitus confirmed the hyperglycemia and glucosuria states. Point the available value of glucose concentration in the blood plasma of this patient:

A. 2,54 mmol/lB. 3,88 mmol/lC. 9,32 mmol/lD. 6,55 mmol/lE. 4,89 mmol/l

Glycogen polysaccharide is synthesized from the active form of glucose. The immediate donor of glucose residues during the glycogenesis is:

A. ADP-glucoseB. Glucose-3-phosphateC. UDP-glucoseD. Glucose-1-phosphateE. Glucose-6-phosphate

Insulin decreases the rate of gluconeogenesis by suppressing of one the following enzymes. Point out this enzyme:

A. HexokinaseB. PhosphofructokinaseC. Pyruvate kinaseD. Glycogen synthaseE. Pyruvate carboxylase

Point out the main process maintaining the blood glucose level during fasting (8–12 hours from person’s last meal):

A. Glycogenolysis in the liverB. Gluconeogenesis in the musclesC. GlycolysisD. Hexose monophosphate shuntE. Glycogenolysis in the muscles

Point out the enzyme whose deficiency can cause the Gierke's disease development:

A. Glycogen-branching enzymeB. Glycogen phosphorylaseC. Amylo- 1,6 -glycosidaseD. Glucose-6-phosphataseE. Alpha-1,4-glycosidase

Point out the type of bonds between glucose-monomers in glycogen molecule destroyed by glycogen phosphorylase a:

A. Beta(1-4), alpha ?(1-6)-glycosidic bondsB. Only alpha?(1-4)-glycosidic bond

C. ??Alpha ?(1-4),? beta(1-6) -glycosidic bondsD. Beta(1-4)-glycosidic bondE. Alpha?(1-4), alpha ?(1-6)-glycosidic bonds

Blood glucose levels cannot be augmented by mobilization of muscle glycogen due to lack of:

A. AldolaseB. HexokinaseC. Glucose-6-phosphataseD. Glucose-6-phosphate dehydrogenaseE. Glucokinase

Patient with diabetes mellitus experienced loss of consciousness and convulsions after an injection of insulin. What might be the result of biochemical blood analysis for concentration of glucose?

A. 5,5 mmole/LB. 10 mmole/LC. 1,5 mmole/L .D. 3,3 mmole/LE. 8,0 mmole/L

Choose the substance that can be the terminal product of glycogenolysis in muscles:

A. Glucose-6-phosphateB. GlucoseC. FructoseD. PyruvateE. Glycogen

A liver biopsy has been made for child with hepatomegaly and mild fasting hypoglycemia. Hepatocytes contant shows the accumulation of glycogen granules with single glucose residues remaining at the branch points near the periphery of the granule. The most likely genetic disorder in this child is associated with the defect in:

A. PhosphoglucomutaseB. Glucosyl -4:6 transferase (branching enzyme)C. Glycogen phosphorylaseD. Amylo alpha-1,6 glucosidase (debranching enzyme)E. Lysosomal alpha-1,4 glucosidase

What hormone decreases the glucose concentration in the blood, if its value is more than 6.8 mM/L:

A. ThyroxinB. InsulinC. GlucagonD. TestosteroneE. Adrenalin

Which of the following statements about glycogen metabolism is true?

A. Cyclic AMP–activated protein kinase stimulates glycogen synthaseB. Epinephrine and glucagon can low cAMP levels in cytoplasma thus stimulating glycogenolysisC. Glycogen phosphorylase b is inactivated by phosphorylationD. Glycogen synthesis is stimulated by glucagonE. Glycogen phosphorylase kinase is activated by phosphorylation

A 50-year old patient with food poisoning is on a drip of 10 % glucose solution. It does not provide the body with necessary energy only, but also performs the function of detoxification by the production of a metabolite that participates in the following conjugation reaction:

A. SulfationB. GlucuronidationC. Glycosylation

D. HydroxylationE. Methylation

Point out the enzyme that breaks beta1-->4 linkages in hyaluronic acid and other glycosaminoglycans?

A. LactaseB. HyaluronidaseC. CellulaseD. Glycogen synthaseE. Glycogen phosphorylase

Find out the pair of enzymes providing the reciprocal control of carbohydrate metabolic pathways:

A. Branching and debranching enzymesB. Acetyl CoA carboxylase and pyruvate kinaseC. Phosphohexose isomerase ang phosphotriose isomeraseD. Hexokinase and glucose 6-phosphataseE. Glycogen synthase and glycogen phosphorylase